ma13 { geometria { av1 { 2014 -...

97
MA13 – Geometria – AV1 – 2014 Quest˜ ao 1 [ 2,0 pt ] Considere um paralelogramo ABCD e sejam M o centro da circunferˆ encia definida pelos v´ ertices A, B e C N o centro da circunferˆ encia definida pelos v´ ertices B, C e D; O o centro da circunferˆ encia definida pelos v´ ertices C , D e A; P o centro da circunferˆ encia definida pelos v´ ertices D, A e B; com M , N , O e P dois a dois distintos. Nessascondi¸c˜ oes, e sabendo que ABCD ao ´ e um retˆ angulo, mostre que: (a) MNOP ´ e um paralelogramo; (b) MNOP e ABCD possuem ˆ angulos internos iguais. Solu¸c˜ ao (a) Como M ´ e centro de uma circunferˆ encia que cont´ em A e B, temos MA MB, portanto, M pertence ` a mediatriz do lado AB. Mas P tamb´ em ´ e centro de uma circunferˆ encia que cont´ em A e B, logo, P tamb´ em pertence ` a mediatriz do lado AB. Assim, MP est´ a contido nesta mediatriz. Da mesma forma, N e O ao centros de circunferˆ encias que contˆ em os pontos C e D, portanto, N e O pertencem ` a mediatriz do lado CD, o que implica que o segmento NO est´ a contido nesta mediatriz. Como ABCD ao ´ e um retˆ angulo, as mediatrizes que contˆ em MP e NO ao coincidem. Como os lados AB e CD ao paralelos, suas respectivas mediatrizes tamb´ em ser˜ ao. Mas, como os segmentos MP e NO est˜ ao contidos cada um em uma destas duas mediatrizes, eles ser˜ ao tamb´ em paralelos. Por argumenta¸c˜ ao an´ aloga, MN e OP est˜ ao contidos, respectivamente, nas mediatrizes de BC e AD, portanto ser˜ ao tamb´ em paralelos. Isto mostra que MNOP ´ e um paralelogramo.

Upload: phamanh

Post on 21-Jan-2019

381 views

Category:

Documents


3 download

TRANSCRIPT

Page 1: MA13 { Geometria { AV1 { 2014 - professor.ufabc.edu.brprofessor.ufabc.edu.br/~jair.donadelli/MA13/MA13.pdf · determine a soma dos quadrados das medidas das diagonais AC e BD em fun˘c~ao

MA13 – Geometria – AV1 – 2014

Questao 1 [ 2,0 pt ]

Considere um paralelogramo ABCD e sejam

• M o centro da circunferencia definida pelos vertices A, B e C

• N o centro da circunferencia definida pelos vertices B, C e D;

• O o centro da circunferencia definida pelos vertices C, D e A;

• P o centro da circunferencia definida pelos vertices D, A e B;

com M , N , O e P dois a dois distintos. Nessas condicoes, e sabendo que ABCD nao e um retangulo,

mostre que:

(a) MNOP e um paralelogramo;

(b) MNOP e ABCD possuem angulos internos iguais.

Solucao

(a) Como M e centro de uma circunferencia que contem A e B, temos MA ≡ MB, portanto, M pertence a

mediatriz do lado AB. Mas P tambem e centro de uma circunferencia que contem A e B, logo, P tambem

pertence a mediatriz do lado AB. Assim, MP esta contido nesta mediatriz.

Da mesma forma, N e O sao centros de circunferencias que contem os pontos C e D, portanto, N e O pertencem

a mediatriz do lado CD, o que implica que o segmento NO esta contido nesta mediatriz.

Como ABCD nao e um retangulo, as mediatrizes que contem MP e NO nao coincidem. Como os lados AB

e CD sao paralelos, suas respectivas mediatrizes tambem serao. Mas, como os segmentos MP e NO estao

contidos cada um em uma destas duas mediatrizes, eles serao tambem paralelos. Por argumentacao analoga,

MN e OP estao contidos, respectivamente, nas mediatrizes de BC e AD, portanto serao tambem paralelos.

Isto mostra que MNOP e um paralelogramo.

Page 2: MA13 { Geometria { AV1 { 2014 - professor.ufabc.edu.brprofessor.ufabc.edu.br/~jair.donadelli/MA13/MA13.pdf · determine a soma dos quadrados das medidas das diagonais AC e BD em fun˘c~ao

(b) Denotemos por E e F os pontos medios de AB e BC, respectivamente.

Considerando o quadrilatero MEBF , temos

∠(EBF ) + ∠(BFM) + ∠(FME) + ∠(MEB) = 360◦,

e, como BFM e MEB sao retos, temos

∠(ABC) + ∠(FME) + ∠(EBF ) + ∠(FME) = 180◦.

Por outro lado,

∠(ONM) + ∠(FME) = ∠(ONM) + ∠(NMP ) = 180◦,

pois ONM e NMP sao angulos suplementares, ja que MP e OP sao retas paralelas.

Assim, ∠(ABC) = ∠(ONM), valendo o mesmo para seus congruentes no paralelogramo, isto e, ∠(ADC) =

∠(MPO). Consequentemente, seus suplementares BCD, DAB, NMP e PON serao tambem congruentes.

Questao 2 [ 2,0 pt ]

Seja ABCD (em ordem) um quadrilatero convexo. Prove que se

∠BDA = ∠ACB

entao o quadrilatero e inscritıvel em um cırculo. Verifique, justificando, se a recıproca e verdadeira.

Solucao

Considere o cırculo determinado por A, B e C. Vamos provar que D pertence a esta circunferencia. Seja D′ a

intersecao (diferente de B) da semirreta−−→BD com a circunferencia. Supondo por absurdo que D nao pertenca a

circunferencia, teremos D′ 6= D. Com isso, ∠(BD′A) = ∠(ACB), pois ambos sao angulos inscritos na circunferencia

e determinam o mesmo arco_

AB, e, com isso, ∠(BD′A) = ∠(BDA), pois ∠(BDA) = ∠(ACB) por hipotese.

Page 3: MA13 { Geometria { AV1 { 2014 - professor.ufabc.edu.brprofessor.ufabc.edu.br/~jair.donadelli/MA13/MA13.pdf · determine a soma dos quadrados das medidas das diagonais AC e BD em fun˘c~ao

Caso D esteja entre D′ e B, o angulo externo ADB do triangulo ADD′ seria congruente ao angulo interno AD′D,

o que e um absurdo. Caso D′ esteja entre D e B, o angulo externo AD′B seria congruente ao angulo interno ADD′,

o que tambem e um absurdo.

A recıproca e obviamente verdadeira, pois se ABCD e inscritıvel em um cırculo, os angulos inscritos BDA e ACB

determinarao o mesmo arco_

AB, portanto, ∠(BDA) = ∠(ACB).

Questao 3 [ 2,0 pt ]

Num cırculo, tomam-se um diametro AB e uma corda CD, perpendiculares entre si. Sendo E um ponto

qualquer de CD, as retas suportes de AE e BE cortam a circunferencia em F e G, respectivamente.

(a) Prove que a semirreta−→FA bissecta CFD e que a semirreta

−−→GB bissecta CGD.

(b) Prove que o quadrilatero CFDG tem dois lados consecutivos na mesma razao que os dois outros.

Solucao

(a) Como AB e um diametro que intersecta a corda CD perpendicularmente, esta intersecao ocorrera no ponto

medio M de CD. Assim, sendo O o centro do cırculo, pelo caso LLL, os triangulos COM e DOM serao

congruentes, e entao COA ≡ DOA.

Com isso,_

AC≡_

AD, portanto os angulos inscritos que os determinam, CFA e AFD, serao congruentes. Isto

mostra que a semirreta−→FA intersecta o angulo CFD.

Da mesma forma,_

BC≡_

BD, logo os angulos incritos, CGB e DGB, serao congruentes. Isto mostra que a

semirreta−−→GB intersecta o angulo CGD.

(b) Considerando o triangulo CFD, FE sera uma bissetriz interna, portanto

CE

CF=

DE

DF∴

DF

CF=

DE

CE.

Da mesma forma, no triangulo CGD, GE tambem sera uma bissetriz interna, e entao

CE

CG=

DE

DG∴

DG

CG=

DE

CE.

Assim,DF

CF=

DG

CG.

Page 4: MA13 { Geometria { AV1 { 2014 - professor.ufabc.edu.brprofessor.ufabc.edu.br/~jair.donadelli/MA13/MA13.pdf · determine a soma dos quadrados das medidas das diagonais AC e BD em fun˘c~ao

Questao 4 [ 2,0 pt ]

Dado um triangulo acutangulo ABC de baricentro G, constroi-se os triangulos equilateros ABD e ACE,

exteriores a ABC, de baricentros H e I, respectivamente.

(a) Mostre que CD ≡ BE.

(b) Mostre que GH ≡ GI.

Solucao

(a) Observe que ∠(CAD) = ∠(CAB)+60◦ = ∠(EAB). Alem disso, como AC ≡ AE e AD ≡ AB, pelo caso LAL,

os triangulos ACD e AEB serao congruentes. Com isso, os lados BE e CD serao congruentes.

(b) Sejam M e N pontos medios de AB e AC, respectivamente.

Como

GN =1

3BN e IN =

1

3EN,

e como GNI ≡ BNE, os triangulos BNE e GNI serao semelhantes, de razao de semelhanca 13 . Assim,

GI =1

3BE.

Analogamente, trabalhando com os triangulos semelhantes CMD e GMH,

GH =1

3CD.

Mas, como CD ≡ BE, temos GI = GH, portanto GI ≡ GH.

Page 5: MA13 { Geometria { AV1 { 2014 - professor.ufabc.edu.brprofessor.ufabc.edu.br/~jair.donadelli/MA13/MA13.pdf · determine a soma dos quadrados das medidas das diagonais AC e BD em fun˘c~ao

Questao 5 [ 2,0 pt ]

Seja ABCD um trapezio de base maior AB e base menor CD. Sendo AB = a, CD = c, BC = b, AD = d,

determine a soma dos quadrados das medidas das diagonais AC e BD em funcao de a, b, c e d. Solucao

Sejam por P e Q as projecoes ortogonais sobre AB de C e D, respectivamente. Denotemos por m e n as medidas de

PB e AQ, respectivamente, e por h a altura do trapezio, relativa a AB.

Aplicando o Teorema de Pitagoras aos triangulos APC e BQD, temos

D21 = (a−m)2 + h2,

D22 = (a− n)2 + h2,

onde D1 e D2 sao as medidas das diagonais AC e BD, respectivamente. Por outro lado, aplicando Pitagoras aos

triangulos AQD e BPC,

h2 = d2 − n2,

h2 = b2 −m2,

logo,

D21 = (a−m)2 + d2 − n2,

D22 = (a− n)2 + b2 −m2.

Somando as duas equacoes, temos

D21 + D2

2 = (a−m)2 + d2 − n2 + (a− n)2 + b2 −m2

= 2a2 − 2am− 2an + b2 + d2

= 2a2 − 2a(m + n) + b2 + d2.

Como m + n = a− c, temos entao

D21 + D2

2 = 2a2 − 2a(a− c) + d2 + b2 = 2ac + b2 + d2.

Page 6: MA13 { Geometria { AV1 { 2014 - professor.ufabc.edu.brprofessor.ufabc.edu.br/~jair.donadelli/MA13/MA13.pdf · determine a soma dos quadrados das medidas das diagonais AC e BD em fun˘c~ao

MA13 – Geometria – AV2 – 2014

Questao 1 [ 2,0 pt ]

Na figura a seguir temos que ∠BAC = x/2, ∠BAD = y/2, medidos em radianos, e AB = 2.

Com base nessas informacoes:

(a) Expresse a area dos triangulos ABC e ABD como funcoes de x e y.

(b) Mostre que

Area (ABD)

Area (ABC)< 1 +

Area (AED)

Area (ABE).

(c) Mostre que para 0 < x < y < π/2 vale

sin(x)

x>

sin(y)

y.

Solucao

(a) Como AB = 2, considerando o triangulo ABC, retangulo em C, temos

BC

2=BC

AB= sen (∠BAC) = sen

(x2

),

logo

BC = 2sen(x

2

).

Da mesma forma,AC

2=AC

AB= cos (∠BAC) = cos

(x2

),

logo

AC = 2cos(x

2

).

Assim,

Area(ABC) =BC ·AC

2=

2sen(x2

)· 2cos

(x2

)2

= 2 sen(x

2

)cos(x

2

)= sen(x).

Page 7: MA13 { Geometria { AV1 { 2014 - professor.ufabc.edu.brprofessor.ufabc.edu.br/~jair.donadelli/MA13/MA13.pdf · determine a soma dos quadrados das medidas das diagonais AC e BD em fun˘c~ao

Considerando agora o triangulo ABD, retangulo em D, e ainda utilizando que AB = 2, temos

BD

2=BD

AB= sen (∠BAD) = sen

(y2

),

logo

BD = 2sen(y

2

),

eAD

2=AD

AB= cos (∠BAD) = cos

(y2

),

que implica

AD = 2cos(y

2

).

Entao,

Area(ABD) =BD ·AD

2=

2sen(y2

)· 2cos

(y2

)2

= 2 sen(y

2

)cos(y

2

)= sen(y).

(b) Observe que

Area(ABD) = Area(ABE) + Area(AED)

e

Area(ABC) = Area(ABE) + Area(BCE) > Area(ABE).

Assim,Area(ABD)

Area(ABC)<

Area(ABE) + Area(AED)

Area(ABE)= 1 +

Area(AED)

Area(ABE).

(c) Pelos itens anteriores, temos

sen(y)

sen(x)=

Area(ABD)

Area(ABC)< 1 +

Area(AED)

Area(ABE).

O setor circular de centro A e determinado pelo arco_

FE esta contido no triangulo ABE. Como a medida de_

FE em radianos e x/2, temos

Area(ABE) >rx

2,

onde r e o raio do cırculo.

O setor circular de centro A e determinado pelo arco_

EG contem o triangulo AED. Como a medida de_

EG

em radianos e y/2− x/2 = (y − x)/2, temos

Area(AED) <r(y − x)

2.

Assim,

sen(y)

sen(x)< 1 +

Area(AED)

Area(ABE)< 1 +

r(y−x)2rx2

= 1 +y − xx

= 1 +y

x− 1 =

y

x,

portantosen(x)

x>

sen(y)

y.

Questao 2 [ 2,0 pt ]

Considere tres retas r, s e t do espaco tais que qualquer plano seja concorrente a pelo menos uma destas

retas. Considere ainda um poliedro tal que

• todas as suas faces sao quadrilateros;

Page 8: MA13 { Geometria { AV1 { 2014 - professor.ufabc.edu.brprofessor.ufabc.edu.br/~jair.donadelli/MA13/MA13.pdf · determine a soma dos quadrados das medidas das diagonais AC e BD em fun˘c~ao

• cada uma de suas arestas e paralela a alguma das retas r, s ou t; e

Prove que todas as faces deste poliedro sao paralelogramos.

Solucao

Considere uma das faces do poliedro, a qual chamaremos de ABCD. Sem perda de generalidade, digamos que

AB ‖ r. Nao poderemos ter BC ‖ r, pois, neste caso, teremos AB e BC consecutivos e colineares. Digamos entao,

sem perda de generalidade, que BC ‖ s.Observe que ja concluımos que cada uma das retas r e s e paralela ou esta contida no plano da face ABCD. Assim,

pela informacao dada sobre as retas, t deve ser concorrente a tal plano, logo nao pode ser paralela a este plano nem

pode estar contida nele. Portanto, nenhuma das arestas CD e AD desta face pode ser paralela a reta t.

Nao poderemos ter CD ‖ s (pois, nesse caso, BC e CD seriam colineares) nem, como ja vimos, CD ‖ t. Portanto,

CD ‖ r. Da mesma forma, AD nao pode ser paralela a r, logo AD ‖ s.

Ate aqui, ja vimos que ABCD e um paralelogramo. Mas a mesma argumentacao vale para qualquer uma das faces

do poliedro, portanto todas as suas faces sao paralelogramos.

Questao 3 [ 2,0 pt ]

Sabendo que a diagonal de um pentagono regular mede 1+√5

2 de seu lado, determine o cosseno do angulo

entre duas faces adjacentes de um icosaedro regular.

Solucao

A partir de um vertice A do icosaedro, podemos retirar uma piramide de base BCDEF pentagonal regular, cujas

faces laterais sao cinco triangulos equilateros que sao faces do icosaedro. Se tomarmos o ponto medio M de AC, o

angulo entre as faces ABC e ACD sera dado por ∠BMD.

Denotando por l a medida da aresta do icosaedro, como BM e DM sao alturas de triangulos equilateros de lado

l, teremos

BM = DM =l√

3

2.

Como BD e uma diagonal do pentagono regular BCDEF , de lado l, temos

BD =1 +√

5

2l.

Assim, pela lei dos cossenos, fazendo ∠BMD = θ, temos(1 +√

5

2l

)2

=

(l√

3

2

)2

+

(l√

3

2

)2

− 2

(l√

3

2

)(l√

3

2

)cos θ,

Page 9: MA13 { Geometria { AV1 { 2014 - professor.ufabc.edu.brprofessor.ufabc.edu.br/~jair.donadelli/MA13/MA13.pdf · determine a soma dos quadrados das medidas das diagonais AC e BD em fun˘c~ao

logo3 +√

5

2=

3

2− 3

2cos θ

e entao

cos θ = −√

5

3.

Assim, o angulo entre as faces e

θ = arccos

(−√

5

3

).

Questao 4 [ 2,0 pt ] Questao anulada

Um poliedro convexo tem exatamente 3 faces triangulares, 1 face quadrangular, 1 face pentagonal e 2 faces

hexagonais. Obtenha:

(a) O numero total de vertices, faces e arestas do poliedro.

(b) O numero de diagonais do poliedro

(c) A soma dos angulos internos de todas as faces.

Solucao

A questao esta anulada. O poliedro descrito no enunciado nao existe; vejamos o motivo.

Somando o numero de faces de cada tipo, o poliedro tera 3+1+1+2 = 7 faces. Multiplicando o numero de arestas

em cada tipo de face pelo numero de faces de cada tipo, e somando os resultados obtidos, estaremos “contando” duas

vezes cada aresta, assim,

2A = 3 · 3 + 1 · 4 + 1 · 5 + 2 · 6 = 9 + 4 + 5 + 12 = 30,

logo A = 15. Como

V −A+ F = 2,

temos

V − 15 + 7 = 2, logo V = 10.

Por outro lado (veja Problema 2.5 do Capıtulo 9),

30 = 2A = 3V3 + 4V4 + 5V5 + ...,

onde Vi e o numero de vertices nos quais incidem i arestas.

Assim,

30 = 3V3 + 4V4 + 5V5 + ... ≥ 3V3 + 3V4 + 3V5 + ... =

= 3(V3 + V4 + V5 + ...) = 3V = 30.

Com isso, a desigualdade e, na verdade, uma igualdade, e, portanto,

3V3 + 4V4 + 5V5 + ... = 3V3 + 3V4 + 3V5,

Page 10: MA13 { Geometria { AV1 { 2014 - professor.ufabc.edu.brprofessor.ufabc.edu.br/~jair.donadelli/MA13/MA13.pdf · determine a soma dos quadrados das medidas das diagonais AC e BD em fun˘c~ao

o que so e possıvel se V4 = V5 = ... = 0, logo V3 = V = 10. Assim, o poliedro possui apenas vertices nos quais

incidem 3 arestas.

De posse dessa informacao, vamos tentar construir o poliedro.

Se as duas faces hexagonais nao tiverem vertice comum (figura 1), o poliedro devera ter pelo menos 12 vertices.

Mas isto contraria V = 10.

Se as duas faces hexagonais tiverem apenas um vertice comum (figura 2), o poliedro devera ter pelo menos 11

vertices, contrariando tambem V = 10.

Assim, como duas faces nao podem ter tres ou mais dois vertices em comum, as faces hexagonais terao exatamente

dois vertices em comum (figura 3). Note que os vertices destas duas faces ja sao todos os 10 vertices que o poliedro

pode ter.

No vertice A acima nao pode incidir nenhuma outra aresta (pois nele ja incidem as 3 arestas possıveis). Portanto,

devemos ter A, F e J em uma mesma face. Por outro lado, esta face nao pode ter nenhum dos outros pontos como

vertice, pois, nesse caso, ela teria tres vertices em comum com alguma das faces hexagonais. Assim, AFJ sera uma

face (Figura 4).

Da mesma forma, BCG tambem e uma face (Figura 4).

Repare que F , J , C e G ja “esgotaram” as arestas que neles incidem.

O segmento FJ e aresta de alguma face, mas como nao e possıvel incidir mais alguma aresta em F e J , esta

face devera ter como vertices os pontos E, F J e I. Veja que esta face ja tem dois vertices em comum com cada

uma das duas faces hexagonais, logo nao podera ter nenhum outro vertice. Assim, tal face e EFJI, logo uma face

quadrangular (Figura 5).

Da mesma forma, devemos ter obrigatoriamente a face DCGH (Figura 5), o que e um absurdo pois, pelo enunciado,

nao podemos ter mais de uma face quadrangular.

Questao 5 [ 2,0 pt ]

O solido da figura e limitado pelo triangulo ABC, pela lateral de um cone de vertice A e por um segmento

Page 11: MA13 { Geometria { AV1 { 2014 - professor.ufabc.edu.brprofessor.ufabc.edu.br/~jair.donadelli/MA13/MA13.pdf · determine a soma dos quadrados das medidas das diagonais AC e BD em fun˘c~ao

circular de centro O. Sabe-se que O e a projecao ortogonal de A sobre o plano que contem o cırculo

representado, que o angulo BOC e reto e que OA = 6cm e OB = 3cm. Determine o volume do solido.

Solucao

Como o angulo BOC e reto, o solido em questao e obtido retirando-se a piramide de base BOC e vertice A de um

quarto do cone circular reto, cuja base e o cırculo e cujo vertice e A. O volume do quarto de cone sera dado, em

unidades de volume, por

Vc =1

4

(π ·OB2 ·OA

3

)=

1

4· π · 3

2 · 63

=9π

2.

O volume da piramide cuja base e o triangulo retangulo BOC e cuja altura e OA e dado, em unidades de volume,

por

Vp =BO·CO

2 ·OA3

=92 · 6

3= 9.

Assim, o volume procurado e, em unidades de volume,

V = Vc − Vp =9π

2− 9 = 9

(π2− 1).

Page 12: MA13 { Geometria { AV1 { 2014 - professor.ufabc.edu.brprofessor.ufabc.edu.br/~jair.donadelli/MA13/MA13.pdf · determine a soma dos quadrados das medidas das diagonais AC e BD em fun˘c~ao

MA13 – Geometria – AV3 – 2014

Questao 1 [ 2,0 pt ]

Sejam PT e PU segmentos tangentes a duas circunferencias concentricas, com T pertencente a menor e U

a maior. Se o segmento PT corta a circunferencia maior no ponto Q, mostre que

PT2 − PU2

= QT2.

Solucao

Prolongue QT na direcao de T ate o ponto R da circunferencia maior. Sendo O o centro de ambos os cırculos, como

o raio OT e perpendicular a QR, temos que QT = TR A potencia de P em relacao ao cırculo maior e dada tanto

por PU2

quanto por PQ · PR, logo

PU2

= PQ · PR.

Mas PQ = PT −QT e PR = PT + TR = PT +QT . Assim,

PU2

= PQ · PR =(PT −QT

) (PT +QT

)= PT

2 −QT 2.

Portanto,

PT2 − PU2

= QT2.

Solucao alternativa

Como PT e PU sao tangentes as circunferencias, serao retos os angulos PTO e PUO. Assim, aplicando o Teorema

de Pitagoras nos triangulos QTO, PTO e PUO, temos

QO2

= TO2

+QT2, (1)

PO2

= TO2

+ PT2, (2)

PO2

= UO2

+ PU2. (3)

Page 13: MA13 { Geometria { AV1 { 2014 - professor.ufabc.edu.brprofessor.ufabc.edu.br/~jair.donadelli/MA13/MA13.pdf · determine a soma dos quadrados das medidas das diagonais AC e BD em fun˘c~ao

Reescrevendo (1), temos

TO2

= QO2 −QT 2

. (4)

Por (2) e (3), temos

TO2

+ PT2

= UO2

+ PU2,

logo, substituindo a expressao para TO obtida em (4), obtemos

QO2 −QT 2

+ PT2

= UO2

+ PU2.

Como UO e QO sao raios de uma mesma circunferencia, e, portanto, UO = QO, a expressao acima nos da

−QT 2+ PT

2= PU

2,

que, reescrevendo, fica

PT2 − PU2

= QT2.

Questao 2 [ 2,0 pt ]

Considere um polıgono P circunscrito a um cırculo C. Se uma reta r passa pelo centro de C e divide P em

dois polıgonos, P1 e P2, prove que P1 e P2 tem mesma area se, e somente se, tem o mesmo perımetro.

Solucao

Sejam P e Q os pontos onde a reta r intersecta o polıgono e denote por

• P,A1, . . . , An, Q os vertices do polıgono P1 e

• P,B1, . . . , Bm, Q os vertices do polıgono P2,

como na figura abaixo.

Page 14: MA13 { Geometria { AV1 { 2014 - professor.ufabc.edu.brprofessor.ufabc.edu.br/~jair.donadelli/MA13/MA13.pdf · determine a soma dos quadrados das medidas das diagonais AC e BD em fun˘c~ao

Denotemos ainda

• l1 = PA1, l2 = A1A2, . . ., ln = An−1An e ln+1 = AnQ e

• l′1 = PB1, l′2 = B1B2, . . ., l′m = Bm−1Bm e l′m+1 = BmQ.

Observe que o perımetro de P1 e dado por

2P1 = l1 + l2 + . . .+ ln + ln+1 + PQ,

e o de P2 e dado por

2P2 = l′1 + l′2 + . . .+ l′m + l′m+1 + PQ.

Decompondo o polıgono P1 em n+ 1 triangulos de altura dada pelo raio r do cırculo, que tenham o centro O do

cırculo como um dos vertices (figura), a area de P1 e dada por

Area(P1) =l1 · r

2+l2 · r

2+ . . .+

ln · r2

+ln+1 · r

2

=(l1 + l2 + . . .+ ln−1 + ln)r

2

=(2P1 − PQ)r

2.

Da mesma forma, decompondo o polıgono P2 em m+ 1 triangulos, temos

Area(P2) =l′1 · r

2+l′2 · r

2+ . . .+

l′m · r2

+l′m+1 · r

2

=(l′1 + l′2 + . . .+ l′m−1 + l′m)r

2

=(2P2 − PQ)r

2.

Assim,

Area(P1) = Area(P2)⇔ (2P1 − PQ)r

2=

(2P2 − PQ)r

2⇔ 2P1 = 2P2.

Page 15: MA13 { Geometria { AV1 { 2014 - professor.ufabc.edu.brprofessor.ufabc.edu.br/~jair.donadelli/MA13/MA13.pdf · determine a soma dos quadrados das medidas das diagonais AC e BD em fun˘c~ao

Questao 3 [ 2,0 pt ]

Sejam ABCD um quadrado de lado L, a semicircunferencia de diametro CD, o segmento BG tangente a

semicircunferencia em E, conforme a figura abaixo. Calcule, em funcao de L, a medida do segmento DG.

Solucao

Seja F o centro da semicircunferencia. Como BE ≡ BC, e FE ≡ FC, os triangulos BEF e BCF serao congruentes

pelo caso LLL, logo BFE ≡ BFC.

Da mesma forma, serao congruentes os triangulos GEF e GDF . Logo,

GFE ≡ GFD e DGF ≡ EGF.

Como

180◦ = ∠(DFE) + ∠(EFC) = 2∠(GFD) + 2∠(BFC),

temos

∠(GFD) + ∠(BFC) = 90◦,

e, como ∠(CBF ) + ∠(BFC) = 90◦, temos GFD ≡ CBF . Assim, os triangulos retangulos BCF e FDG sao

semelhantes, comBC

FD=CF

DG∴LL2

=L2

DG∴ DG =

L

4

Page 16: MA13 { Geometria { AV1 { 2014 - professor.ufabc.edu.brprofessor.ufabc.edu.br/~jair.donadelli/MA13/MA13.pdf · determine a soma dos quadrados das medidas das diagonais AC e BD em fun˘c~ao

Questao 4 [ 2,0 pt ]

Um plano e perpendicular a diagonal AG do cubo ABCDEFGH da figura, de forma que sua intersecao

com as faces do cubo seja o hexagono UVWXY Z.

(a) Mostre que cada lado do hexagono UVWXY Z e paralelo a uma das diagonais da face do cubo em

que esta contido.

(b) Determine o perımetro do hexagono UVWXY Z, sendo 1 a medida da aresta do cubo.

Solucao

(a) Primeiramente, observe que a diagonal AG do cubo e ortogonal as diagonais de face ED, BD e BE, pois AG

contem a altura do tetraedro AEDB. Da mesma forma, AG e ortogonal as diagonais CH, CF e FH.

O plano π considerado, perpendicular a diagonal AG, e paralelo ao plano determinado por E, D e B, portanto, a

intersecao Y Z com a face ADHE e paralela a diagonal ED desta face. Analogamente, UV e WX sao paralelos

as diagonais BD e BE das faces em que estao.

Da mesma forma, π e paralelo ao plano determinado por C, F e H, portanto, UZ, VW e XY serao paralelos,

respectivamente, as diagonais de face CH, CF e FH.

(b) Pelo item (a), serao isosceles os triangulos CUV , BVW , FXW , EXY , HY Z e DUZ. Fazendo HY = a,

teremos entao

a = HY = HZ = UC = CV = WF = XF,

1− a = EY = DZ = DU = BV = BW = EX.

Assim,

Y Z = UV = WX = a√

2,

UZ = VW = XY = (1− a)√

2.

Logo, o perımetro de UVWXY Z e dado por

Y Z + UV +WX + UZ = VW = XY = 3a√

2 + 3(1− a)√

2 = 3√

2.

Page 17: MA13 { Geometria { AV1 { 2014 - professor.ufabc.edu.brprofessor.ufabc.edu.br/~jair.donadelli/MA13/MA13.pdf · determine a soma dos quadrados das medidas das diagonais AC e BD em fun˘c~ao

Questao 5 [ 2,0 pt ]

Considere o cubo ABCDEFGH de aresta a. Um cone C1 tem base inscrita na face ABCD e vertice na

interseccao das diagonais da face EFGH. Outro cone C2 tem base inscrita na face EFGH e vertice na

interseccao das diagonais da face ABCD. Calcule o volume da parte comum a esses dois cones.

Solucao

Seja I o ponto na face ABCD que e vertice de um dos cones, e J o ponto na face EFGH, vertice do outro cone.

Considere os pontos K e M , intersecao de cada um dos cones com as arestas AB e EF , respectivamente (veja a

figura).

Como K e M sao pontos medios das arestas em que estao, e I e J sao os centros de suas faces, IJMK e um

retangulo, cujo centro e o ponto L, intersecao das geratrizes IM e JK dos cones. A distancia de L ao centro O do

cubo sera dada entao por KI/2 = a/4. A distancia de O a cada um dos pontos I e J sera igual a a/2.

Assim, a intersecao dos cones sera um solido formado por dois cones, cuja base e um cırculo de raio OL = a/4, e

cujas alturas OI e OJ medem a/2. Portanto, o volume deste solido e dado por

V = 2

(π(a4

)2 · a23

)=π a3

48.

Page 18: MA13 { Geometria { AV1 { 2014 - professor.ufabc.edu.brprofessor.ufabc.edu.br/~jair.donadelli/MA13/MA13.pdf · determine a soma dos quadrados das medidas das diagonais AC e BD em fun˘c~ao

MA13 – Geometria – AVF – 2014

Questao 1 [ 2,0 pt ]

Na figura, AB ≡ AC e a bissetriz interna tracada de B intersecta o lado AC em P de forma que AP + BP = BC.

Os pontos Q e D sao tomados de forma que BQ ≡ BP e PD e paralelo a BC.

(a) Mostre que os triangulos CQP e PAD sao congruentes.

(b) Determine as medidas dos angulos do triangulo ABC.

Solucao

(a) Para simplificar a notacao, denote ∠(CBP ) = β. Como BP e bissetriz de ABC, temos tambem ∠(ABP ) = β. E, como

AB ≡ AC, temos ∠(ACB) = ∠(ABC) = 2β.

Como BQ+QC = BC = BP + PA e BQ = BP , temos QC = AP .

Como BC e DP sao paralelas, temos

∠(BPD) = ∠(CBP ) = β,

∠(APD) = ∠(ACB) = 2β.

Como ADP e angul oexterno do triangulo BDP , temos

∠(ADP ) = ∠(BPD) + ∠(DBP ) = 2β.

Mas entao o triangulo ADP sera isosceles, e, portanto, AD ≡ AP . Como AB ≡ AC, temos que BD ≡ PC, e, como

BDP e isosceles, temos DP ≡ BD ≡ CP .

Pelo caso LAL, temos entao que os triangulos PAD e CQP sao congruentes.

Page 19: MA13 { Geometria { AV1 { 2014 - professor.ufabc.edu.brprofessor.ufabc.edu.br/~jair.donadelli/MA13/MA13.pdf · determine a soma dos quadrados das medidas das diagonais AC e BD em fun˘c~ao

(b) Pela congruencia de PAD e CQP , temos ∠(QPC) = ∠(ADP ) = 2β.

Como PQC e isosceles, ∠(QPB) = ∠(PQB), e como ∠(QPB) + ∠(PQB) + β = 180◦, temos ∠(QPB) = 90◦ − β2

.

Assim, considerando todos os angulos da figura que tem vertice em P , temos

2β +

(90◦ − β

2

)+ β + 2β = 180◦,

logo9

2β = 90◦ ∴ β = 20◦.

com isso, os angulos internos B e C medem 2β = 40◦, e A mede 180◦ − 2 · 40◦ = 100◦.

Questao 2 [ 2,0 pt ]

Prove que se um trapezio isosceles tem os lados congruentes com comprimento a, os lados paralelos com comprimentos

b e c, e diagonais com comprimento d, entao d2 = a2 + bc.

Solucao

Vamos supor, sem perda de generalidade, b > c. Denotemos por h a altura do trapezio e por m a projecao ortogonal de um

dos lados congruentes sobre a base maior. Note que m = b−c2

.

Assim, considerando um triangulo retangulo cujos catetos sao m e h, e cuja hipotenusa e um lado a, temos, pelo Teorema

de Pitagoras,

m2 + h2 = a2,

logo (b− c

2

)2

+ h2 = a2,

que nos da

b2 + c2 − 2bc+ 4h2 = 4a2.

Considerando agora um triangulo retangulo cujos catetos medem c+m e h, e de hipotenusa d, temos

(c+m)2 + h2 = d2,

logo (c+

b− c

2

)2

+ h2 = d2,

Page 20: MA13 { Geometria { AV1 { 2014 - professor.ufabc.edu.brprofessor.ufabc.edu.br/~jair.donadelli/MA13/MA13.pdf · determine a soma dos quadrados das medidas das diagonais AC e BD em fun˘c~ao

e entao (b+ c

2

)+ h2 = d2,

que implica

b2 + c2 + 2bc+ 4h2 = 4d2.

Subtraindo as duas equacoes obtidas, temos

4bc = 4d2 − 4a2,

logo,

d2 = a2 + bc.

Questao 3 [ 2,0 pt ]

Na figura abaixo o triangulo ABC e retangulo em A. Os quadrilateros ABED e ACGF sao quadrados. Estendemos

EB ate P , de tal modo que EB ≡ BP . Estendemos GC ate Q, de tal modo que GC ≡ CQ.

(a) Prove que o triangulo ABC e congruente ao triangulo PBI e que o triangulo BQC e congruente ao triangulo

HPI.

(b) Prove que a area do triangulo BPC e a metade da area do quadrado ABED.

(c) Prove que a area do triangulo BQC e a metade da area do quadrado ACGF .

(d) Demonstre que AB2

+ AC2

= BC2

(Teorema de Pitagoras).

Solucao

(a) Por simplicidade, vamos denotar ∠(ABC) = β e ∠(ACB) = γ. Note que β + γ = 90◦.

Como ∠(ABC) = β e ABC e angulo reto, ∠(PBC) = 90◦−β = γ. E, como CBI e angulo reto, ∠(PBI) = 90◦−γ = β.

Assim, temos PB ≡ EB ≡ AB, BI ≡ BC e ∠(PBI) = ∠(ABC), logo, pelo caso LAL, os triangulos ABC e PBI sao

congruentes.

Essa congruencia implica que ∠(PIB) = ∠(ACB) = γ, logo ∠(PIH) = 90 − γ = β, e que PI ≡ AC.

Temos ∠(BCQ) = 90◦ − γ, logo, ∠(BCQ) = β.

Note ainda que QC ≡ CG ≡ AC.

Page 21: MA13 { Geometria { AV1 { 2014 - professor.ufabc.edu.brprofessor.ufabc.edu.br/~jair.donadelli/MA13/MA13.pdf · determine a soma dos quadrados das medidas das diagonais AC e BD em fun˘c~ao

Temos entao que

∠(PIH) = β = ∠(BCQ),

P I ≡ AC ≡ QC,

IH ≡ CB.

Com isso os triangulos PBI e BQP serao congruentes pelo caso LAL.

(b) Como AC e paralelo a BP , temos

Area(BPC) = Area(BPA) =BP ·AB

2=AB

2

2,

que e a metade da area AB2

do quadrado ABED.

(c) Como AB e paralelo a QC, temos

Area(BQC) = Area(AQC) =QC ·AC

2=AC

2

2,

que e a metade da area AC2

do quadrado ACGF .

(d) Pelos itens anteriores, ja sabemos que

AB2

+AC2

= 2Area(BPC) + 2Area(BQC).

Portanto, resta apenas mostrar que 2Area(BPC) + 2Area(BQC) = BC2

Pelo item (a), temos Area(BQC) = Area(PIH), sendo h1 e h2 as alturas dos triangulos BPC e PIH, relativas as bases

BC e IH, respectivamente (veja figura), temos h1 + h2 = CH = BC, logo

2Area(BPC) + 2Area(BQC) = 2Area(BPC) + 2Area(PIH)

= 2 · BC · h1

2+ 2 · IH · h2

2= BC · h1 +BC · h2

= (h1 + h2)BC

= BC2.

Assim,

AB2

+AC2

= 2Area(BPC) + 2Area(BQC) = BC2.

Questao 4 [ 2,0 pt ]

Um tetraedro regular e cortado por um plano paralelo a duas arestas, de tal forma que a secao seja um paralelogramo.

(a) Descreva a posicao do plano de forma que a secao seja um losango e calcule, em funcao de a, o lado desse

losango.

Page 22: MA13 { Geometria { AV1 { 2014 - professor.ufabc.edu.brprofessor.ufabc.edu.br/~jair.donadelli/MA13/MA13.pdf · determine a soma dos quadrados das medidas das diagonais AC e BD em fun˘c~ao

(b) Determine, em funcao da medida a da aresta, a medida do lado do paralelogramo de area maxima assim obtido.

Solucao

(a) Para facilitar a escrita, seja ABCD o tetraedro e consideremos o plano paralelo as arestas AB e CD. Sejam ainda X,

Y , Z e W as intersecoes do plano com as arestas AB, BC, BD e AD, respectivamente.

O segmento XY e a intersecao do plano paralelo a AB com a face ABC, contida em um plano que tambem contem

AB. Ora, intersecao de dois planos paralelos a uma reta dada (ou que contenham esta reta) sera uma reta paralela a

reta dada, portanto, o segmento XY e paralelo a aresta AB. Da mesma forma, o segmento ZW sera paralelo a AB, e

Y Z e XW paralelos a CD.

Como ABC e um triangulo equilatero, e XY e paralelo a AB, temos que XCY e equilatero, implicando XY ≡ CX.

Da mesma forma, como ACD e equilatero e XW e paralelo a CD, temos que XW ≡ AX.

Para que XY ZW seja um losango, e necessario que XY ≡ XW , logo, que

CX ≡ XY ≡ XW ≡ AX.

Assim, X sera o ponto medio de AC. Com isso, como CY ≡ CX e BC ≡ AC, Y tambem sera ponto medio de BC. Da

mesma forma, Z e W sao pontos medios de BD e AD, respectivamente.

(b) Na notacao do item anterior, sendo XC = x, teremos XY = x e XW = AX = a− x. As retas reversas suporte de AB

e CD sao ortogonais, logo, sendo XY e XW paralelas a estas retas, respectivamente, XY e XW sao perpendiculares.

Assim, a area do paralelogramo XY ZW e dada por

Area(XY ZW ) = XC ·XW = x(a− x) = x2 + ax.

O valor maximo para esta expressao ocorre quando x = a2. Portanto, o paralelogramo de area maxima ocorre quando

seus lados medem a2.

Questao 5 [ 2,0 pt ]

Sejam x, y e z os volumes gerados por um triangulo ABC, retangulo em A, girando sucessivamente em torno de seus

lados BC, CA e AB. Prove que1

x2=

1

y2+

1

z2

Solucao

Page 23: MA13 { Geometria { AV1 { 2014 - professor.ufabc.edu.brprofessor.ufabc.edu.br/~jair.donadelli/MA13/MA13.pdf · determine a soma dos quadrados das medidas das diagonais AC e BD em fun˘c~ao

Quando giramos em torno do cateto CA, temos um cone circular reto cuja base e o cırculo de raio AB e de altura CA.

Assim, o volume e

y =πAB

2 · CA3

.

Quando giramos em torno do cateto AB, temos um cone circular reto cuja base e o cırculo de raio CA e de altura AB.

Assim, o volume e

z =πCA

2 ·AB3

.

Girando em torno da hipotenusa BC, obtemos dois cones. A base de ambos sera o cırculo de raio igual a altura h do

triangulo ABC, relativa ao vertice A. As alturas dos cones serao as medidas m e n das projecoes ortogonais de AB e AC

sobre a hipotenusa BC. Assim, o volume do solido dado pelos dois cones e

x =πh2 ·m

3+πh2 · n

3

=πh2(m+ n)

3

=πh2 ·BC

3

Como h ·BC = AB · CA, temos h = AB·CABC

, logo

x =πAB

2 · CA2

3BC.

Assim,

1

y2+

1

z2=

9

π2AB4 · CA2 +

9

π2CA4 ·AB2

=9CA

2+ 9AB

2

π2AB4 · CA4

=9BC

2

π2AB4 · CA4

=

(3BC

πAB2 · CA2

)2

=1

x2.

Page 24: MA13 { Geometria { AV1 { 2014 - professor.ufabc.edu.brprofessor.ufabc.edu.br/~jair.donadelli/MA13/MA13.pdf · determine a soma dos quadrados das medidas das diagonais AC e BD em fun˘c~ao

GABARITO MA13 - Avaliacao 1 - 2o semestre - 2013

Questao 1. (pontuacao: 2)

ABCDE e um pentagono regular e ABF e um triangulo equilatero interior ao pentagono. Calcule os angulos internos

do triangulo AFC.

Uma solucao:

Cada angulo interno do pentagono regular mede

180o(5− 2)

5= 108o = ABC

Sendo AB = BC, entao BAC = BCA =180o − 108o

2= 36o. Assim, FAC = FAB − CAB = 60o − 36o = 24o.

Temos ainda FBC = ABC −ABF = 108o − 60o = 48o.

Sendo BF = BC, entao BFC = BCF =180o − 48o

2= 66o. Assim, ACF = BCF −BCA = 66o − 36o = 30o.

Finalmente, AFC = AFB + BFC = 60o + 66o = 126o.

Conclusao: os angulos internos do triangulo AFC medem 24o, 30o e 126o.

Questao 2. (pontuacao: 2)

No triangulo ABC tem-se AB = c, BC = a e AC = b. A semirreta AD (D ∈ BC) e bissetriz do angulo BAC e o

ponto I e o incentro do triangulo.

a) (1,0) Calcule a razaoIA

IDem funcao dos lados a, b e c.

b) (1,0) Sendo G o baricentro de ABC mostre que, se IG e paralelo a BC, entao a =b + c

2.

Uma solucao:

Page 25: MA13 { Geometria { AV1 { 2014 - professor.ufabc.edu.brprofessor.ufabc.edu.br/~jair.donadelli/MA13/MA13.pdf · determine a soma dos quadrados das medidas das diagonais AC e BD em fun˘c~ao

a) Pelo teorema da bissetriz interna, temosBD

DC=

AB

AC=

c

b. Entao, devido a uma propriedade util das proporcoes,

BD

c=

DC

b=

a

b + c, pois BD + DC = a

Logo BD =ac

b + c.

Sendo I o incentro entao BI e bissetriz do angulo ABC. Pelo mesmo teorema,

IA

ID=

BA

BD=

cac

b + c

=b + c

a

b) Seja M o ponto medio de BC. Como IG e paralelo a BC entao

IA

ID=

b + c

a=

GA

GM=

2

1

(a ultima igualdade e valida pois G e o baricentro de ABC). Logo, a =b + c

2, c.q.d.

Questao 3. (pontuacao: 2)

No triangulo acutangulo ABC tem-se AB = x− 1, BC = x e AC = x + 1.

a) (1,0) Determine todos os valores possıveis de x para que exista um triangulo nas condicoes descritas acima.

b) (1,0) Seja D o ponto do lado BC tal que AB = AD. Calcule o comprimento do segmento DC.

Uma solucao:

a) Note que AC e o maior lado. Para que ABC seja acutangulo, o maior angulo deve ser menor do que 90o. Isto

significa que devemos ter AC2 < AB2 + BC2. De (x + 1)2 < (x− 1)2 + x2 concluımos que x > 4.

(O “caso limite” x = 4 e o conhecido triangulo retangulo 3− 4− 5)

b) Seja AH perpendicular a BC. Como AB = AD, entao H e medio de BD. Sejam AH = h, BH = a e HC = b.

Page 26: MA13 { Geometria { AV1 { 2014 - professor.ufabc.edu.brprofessor.ufabc.edu.br/~jair.donadelli/MA13/MA13.pdf · determine a soma dos quadrados das medidas das diagonais AC e BD em fun˘c~ao

Nos triangulos retangulos AHC e AHB temos:

HC2 = AC2 −AH2 ou seja b2 = (x + 1)2 − h2

HB2 = AB2 −AH2 ou seja a2 = (x− 1)2 − h2

Subtraindo,

b2 − a2 = (x + 1)2 − (x− 1)2 − 4x

ou seja, (b− a)(a + b) = 4x. Como a + b = BC = x entao b− a = DC = 4.

Questao 4. (pontuacao: 2)

O quadrilatero ABCD esta inscrito em uma circunferencia. Seja M o ponto medio do arco CD como mostra a figura.

Os segmentos MA e MB cortam o lado CD em P e Q, respectivamente.

a) (1,0) Mostre que o quadrilatero ABQP e inscritıvel.

b) (1,0) Mostre que os angulos DAQ e PBC sao iguais.

Uma solucao:

Page 27: MA13 { Geometria { AV1 { 2014 - professor.ufabc.edu.brprofessor.ufabc.edu.br/~jair.donadelli/MA13/MA13.pdf · determine a soma dos quadrados das medidas das diagonais AC e BD em fun˘c~ao

a) Sejam a, b, c, d e novamente d os arcos DA, AB, BC, CM e MD como na figura acima. Vamos calcular a

soma de dois angulos opostos do quadrilatero ABQP .

PAB = MAB =arc(BM)

2=

c + d

2

BQP = BQD =arc(DB) + arc(CM)

2=

a + b + d

2

(ja que BQP e angulo externo ao triangulo DQM). Assim

PAB + BQP =a + b + c + d + d

2=

360o

2= 180o

Logo, ABQP e inscritıvel.

b)

DAQ = DAP + PAQ = DAM + PAQ =d

2+ PAQ

e

PBC = PBQ + QBC = PBQ + MBC = PBQ +d

2

Como ABQP e inscritıvel entao PAQ = PBQ. Logo DAQ = PBC, c.q.d.

Questao 5. (pontuacao: 2)

Considere um triangulo ABC inscrito em uma circunferencia de centro O. Os pontos D e E das retas BC e AC,

respectivamente, sao tais que AD e perpendicular a BC e BE e perpendicular a AC. As retas AD e BE cortam-se

em H. Sejam M , N e P os pontos medios dos segmentos AC, AH e AB, respectivamente.

a) (1,0) Mostre que OMNP e um paralelogramo.

b) (1,0) Mostre que em um triangulo qualquer, a distancia do ortocentro a um vertice e o dobro da distancia do

circuncentro ao lado oposto.

Uma solucao:

Page 28: MA13 { Geometria { AV1 { 2014 - professor.ufabc.edu.brprofessor.ufabc.edu.br/~jair.donadelli/MA13/MA13.pdf · determine a soma dos quadrados das medidas das diagonais AC e BD em fun˘c~ao

a) O ponto O e o circuncentro de ABC e, portanto, pertence as mediatrizes dos lados do triangulo. Assim, OM e

perpendicular a AC e OP e perpendicular a AB. Como P e N sao medios de AB e AH entao PN e paralelo a BH

que, por sua vez, e perpendicular a AC. Logo, PN e OM sao paralelos porque sao ambos perpendiculares a AC.

Agora, AD e BE sao alturas do triangulo ABC e seja CF a terceira altura. Logo, CF passa por H, o ortocentro

do triangulo. Repetindo o argumento, como N e M sao pontos medios de AH e AC entao MN e paralelo a CH

que, por sua vez, e perpendicular a AB. Logo, MN e OP sao paralelos porque sao ambos perpendiculares a AB.

Assim, OMNP e um paralelogramo.

b) A distancia do circuncentro O ao lado AC e OM . No triangulo AHB, como P e N sao medios de AB e AH

respectivamente, entao BH e o dobro de PN que, por sua vez, e igual a OM . Assim, BH = 2.OM , como querıamos

demonstrar.

Obs: Naturalmente que essa propriedade vale qualquer que seja o vertice do triangulo.

Page 29: MA13 { Geometria { AV1 { 2014 - professor.ufabc.edu.brprofessor.ufabc.edu.br/~jair.donadelli/MA13/MA13.pdf · determine a soma dos quadrados das medidas das diagonais AC e BD em fun˘c~ao

GABARITO MA13 Geometria I - Avaliacao 2 - 2013/2

Questao 1. (pontuacao: 2)

As retas r, s e t sao paralelas, como mostra a figura abaixo. A distancia entre r e s e igual a 3 e a distancia entre

s e t e igual a 1. O triangulo equilatero ABC possui os vertices A, B e C sobre as retas r, s e t, respectivamente.

Determine o lado do triangulo ABC.

Uma solucao:

Seja AB = BC = CA = 4a. Sendo D o ponto de intersecao da reta s com o lado AC temos, pelo teorema de

Tales, AD = 3a e DC = a .

Vamos calcular a area de ABC de duas formas.

a) A area de um triangulo equilatero de lado 4a e S =(4a)2

√3

4= 4√

3a2.

b) A area de ABC e a soma das areas dos triangulos BDA e BDC que possuem base comum BD = x e alturas 3

e 1, respectivamente. Entao, S =x.3

2+x.1

2= 2x. Assim, 4

√3a2 = 2x.

Que relacao ha entre a e x?

Page 30: MA13 { Geometria { AV1 { 2014 - professor.ufabc.edu.brprofessor.ufabc.edu.br/~jair.donadelli/MA13/MA13.pdf · determine a soma dos quadrados das medidas das diagonais AC e BD em fun˘c~ao

A relacao de Stewart no triangulo ABC com a ceviana AD fornece

16.a2.3a+ 16a2.a = x2.4a+ a.3a.4a

ou seja, x = a√

13.

Como 4√

3a2 = 2x entao 4√

3a2 = 2a√

13, ou seja, a =

√39

6.

Entao o lado do triangulo e AB = 4a =2√

39

3.

Outra solucao

Seja y o lado do triangulo e sejam α e β os angulos que BA e BC fazem com uma reta perpendicular as paralelas,

respectivamente.

Como cosα =3

yentao senα =

√1− 9

y2=

√y2 − 9

y2e como cosβ =

1

yentao senβ =

√1− 1

y2=

√y2 − 1

y2.

Como ∠ABC = 60o entao α+ β = 120o. Assim, cosα.cosβ − senα.senβ = − 12 .

Daı, fazendo as substituicoes, temos

3

y.1

y−

√y2 − 9

y.

√y2 − 1

y= −1

2

Apos as necessarias manipulacoes algebricas encontramos y =

√52

3=

2√

39

3.

Mais uma solucao

Primeiro tracamos duas perpendiculares as retas paralelas, passando por B e C, como na figura:

Page 31: MA13 { Geometria { AV1 { 2014 - professor.ufabc.edu.brprofessor.ufabc.edu.br/~jair.donadelli/MA13/MA13.pdf · determine a soma dos quadrados das medidas das diagonais AC e BD em fun˘c~ao

x

y z

l

Assim x = y + z e, aplicando tres vezes o teorema de Pitagoras, temosx = y + z

l2 = x2 + 1

l2 = y2 + 9

l2 = z2 + 16

Apesar de nao linear, este sistema pode ser facilmente resolvido. De fato,

√l2 − 1 =

√l2 − 9 +

√l2 − 16

Elevando ao quadrado, isolando o termo que aparecera ainda com a raiz quadrada e elevando novamente ao

quadrado, chegamos a solucao l =2√

39

3.

Obs: Ha varias outras solucoes.

Questao 2. (pontuacao: 2)

Um poliedro convexo P possui 8 vertices, apenas uma face pentagonal e todas as outras faces triangulares.

(1,0) a) Determine o numero de faces triangulares de P .

(1,0) b) Determine o numero de diagonais de P .

Uma solucao:

a) Como P possui 8 vertices e uma face pentagonal (5 vertices), entao ha uma face triangular oposta a essa face

pentagonal. Fazendo um desenho e unindo os vertices dessas duas faces formando triangulos, obtemos o poliedro

abaixo.

Page 32: MA13 { Geometria { AV1 { 2014 - professor.ufabc.edu.brprofessor.ufabc.edu.br/~jair.donadelli/MA13/MA13.pdf · determine a soma dos quadrados das medidas das diagonais AC e BD em fun˘c~ao

Como mostra o desenho, ha 9 faces triangulares.

Outra forma de chegar a mesma conclusao:

No poliedro P temos uma face pentagonal e x faces triangulares: F3 = x e F5 = 1. Como 2A = 3F3 + 5F5, temos

2A = 3x+ 5, ou seja,

A =3x+ 5

2

A relacao A+ 2 = F +V fornece3x+ 5

2+ 2 = x+ 1 + 8, pois so ha faces triangulares e uma pentagonal. A solucao

dessa equacao e x = 9. O poliedro P possui 9 faces triangulares.

b) A face pentagonal possui 5 diagonais. Essas diagonais nao sao diagonais de P . O poliedro P possui 8 vertices,

10 faces e 16 arestas. O numero de diagonais de P e

d = C28 − 16− 5 = 7

Questao 3. (pontuacao: 2)

O quadrado ABCD esta contido no plano Π e DE e um segmento perpendicular a Π. Trace os segmentos EA,

EB e EC formando a piramide EABCD. Considere AB = 2 e DE = 4.

Page 33: MA13 { Geometria { AV1 { 2014 - professor.ufabc.edu.brprofessor.ufabc.edu.br/~jair.donadelli/MA13/MA13.pdf · determine a soma dos quadrados das medidas das diagonais AC e BD em fun˘c~ao

(0,5) a) Calcule o cosseno do angulo que a reta BE faz com Π.

(0,5) b) Calcule o cosseno do angulo que o plano EAB faz com Π.

(1,0) c) Calcule o cosseno do angulo entre os semiplanos EBA e EBC.

Uma solucao:

a) O angulo que BE faz com o plano da base da piramide e o angulo DBE = α. Como AB = 2 entao BD = 2√

2

e, no triangulo EDB, retangulo em B temos, pelo teorema de Pitagoras, EB = 2√

6 . Assim,

cosα =BD

BE=

2√

2

2√

6=

1√3

=

√3

3.

b) ED e perpendicular a DA e DA e perpendicular a AB. Entao, pelo teorema das tres perpendiculares, EA e

perpendicular a AB. Assim, o angulo que o plano EAB faz com o plano da base da piramide e o angulo DAE = β.

No triangulo retangulo DAE calculamos AE = 2√

5 e, em seguida,

cosβ =AD

AE=

2

2√

5=

√5

5.

c) O plano que contem AC e e perpendicular a BE corta BE em P . O angulo entre os semiplanos EBA e EBC

e o angulo APC = θ.

Sejam PA = PC = x (BDE e o plano mediador de AC). No triangulo EAB o segmento PA e a altura relativa a

hipotenusa.

Como, AE.AB = EB.PA, temos 2√

5.2 = 2√

6.x, ou seja, x =2√

5√6

. Assim, x2 =10

3.

No triangulo APC, usando a lei dos cossenos relativa ao vertice P , temos:

AC2 = PA2 + PC2 − 2.PA.PC.cos θ

Entao,

(2√

2)2 = x2 + x2 − 2.x.x.cos θ ⇒ 8 = 2x2 − 2x2cos θ ⇒ 4 = x2 − x2cos θ

Logo 4 =10

3− 10

3cos θ ⇒ cos θ = −1

5.

Page 34: MA13 { Geometria { AV1 { 2014 - professor.ufabc.edu.brprofessor.ufabc.edu.br/~jair.donadelli/MA13/MA13.pdf · determine a soma dos quadrados das medidas das diagonais AC e BD em fun˘c~ao

Questao 4. (pontuacao: 2)

A figura abaixo mostra duas circunferencias de raio 1, tangentes entre si e inscritas em uma semicircunferencia de

diametro AB.

A B

(0,5) a) Calcule o comprimento do segmento AB.

(1,5) b) Calcule a area da regiao sombreada.

Uma solucao:

Observando a figura a seguir, sejam: O, o centro da semicircunferencia, OC o raio da semicircunferencia perpen-

dicular a AB, K, o centro da circunferencia da direita, KE e KF , raios dessa circunferencia perpendiculares a OB

e OC respectivamente, e OD, o raio da semicircunferencia que passa por K.

B

a) OEKF e um quadrado de lado 1. Como D e o ponto de tangencia entre a circunferencia da direita e a

semicircunferencia, entao OD = OK +KD =√

2 + 1.

Assim, AB = 2(√

2 + 1).

b) Seja S a area da regiao sombreada. Para calcular a area dessa regiao, transferimos a sua metade da esquerda

da para a regiao EBD como mostra a figura acima. Assim, S e a area sombreada na figura acima, que e igual a area

do quadrante circular OBC subtraıda da area do quadrado OEKF e da area do setor KEDFK.

Page 35: MA13 { Geometria { AV1 { 2014 - professor.ufabc.edu.brprofessor.ufabc.edu.br/~jair.donadelli/MA13/MA13.pdf · determine a soma dos quadrados das medidas das diagonais AC e BD em fun˘c~ao

S =π(√

2 + 1)2

4− 1− 3π

4

Simplificando, obtemos S =

√2π − 2

2.

Questao 5. (pontuacao: 2)

Considere uma esfera de centro O e raio 1, e seja AB um diametro dessa esfera. Um cone de revolucao possui

vertice A e base de centro B e raio 1. A figura abaixo mostra a secao nesses solidos por um plano que contem a reta

AB.

(1,0) a) Mostre que a circunferencia que e a intersecao da superfıcie da esfera com a superfıcie lateral do cone tem

raio igual a4

5.

(1,0) b) Calcule o volume da parte comum entre a esfera e o cone (relacionado com a area sombreada da figura).

Obs: Voce pode usar o resultado do item a) mesmo que nao o tenha demonstrado.

Uma solucao:

Como na figura abaixo, seja BD o raio da base do cone e CE o raio da circunferencia Γ, interseccao da superfıcie

da esfera com a superfıcie lateral do cone.

Page 36: MA13 { Geometria { AV1 { 2014 - professor.ufabc.edu.brprofessor.ufabc.edu.br/~jair.donadelli/MA13/MA13.pdf · determine a soma dos quadrados das medidas das diagonais AC e BD em fun˘c~ao

Se ∠DAB = α entao tanα =BD

BA=

1

2.

O triangulo AEB e retangulo em E. Assim, fazendo EB = b, temos tanα =EB

EAe, portanto, EA = 2b.

O teorema de Pitagoras no triangulo AEB fornece b2 =4

5.

O segmento CE = r e altura relativa a hipotenusa do triangulo AEB. Assim, EB.EA = AB.CE, ou seja,

b.2b = 2.r e entao r = b2 =4

5.

b) A parte comum entre a esfera e o cone dado e formada por um cone de altura CA cuja base e a circunferencia

Γ, reunido com o segmento esferico com base na circunferencia Γ e contendo o ponto B.

Como OE = 1 e CE = 45 , entao CO = 3

5 .

Assim a altura do cone e H = CA = 35 + 1 =

8

5.

A altura do segmento esferico e h = 1− 35 =

2

5.

O volume da parte comum entre a esfera e o cone dado e

V =1

3π.r2.H +

π.h2

3(3R− h)

V =π

3.(

4

5)2.

8

5+π

3.(

2

5)2.(3.1− 2

5) =

π

3.128

125+π

3.

52

125=π

3.180

125

Logo

V =12π

25

.

Page 37: MA13 { Geometria { AV1 { 2014 - professor.ufabc.edu.brprofessor.ufabc.edu.br/~jair.donadelli/MA13/MA13.pdf · determine a soma dos quadrados das medidas das diagonais AC e BD em fun˘c~ao

AV1 - MA 13 - 2011

Questão 1.

A figura abaixo mostra uma sequência de circunferências de centros C1, C2, . . ., Cn com raios r1, r2, . . ., rn, respec-

tivamente, todas tangentes às retas s e t, e cada circunferência, a partir da segunda, tangente à anterior.

s

t

C1

C2C3

Considere r1 = a e r2 = b.

(1,0) (a) Calcule r3 em função de a e b.

(1,0) (b) Calcule rn em função de a e b.

UMA SOLUÇÃO

s

t

C1

C2C3

AB

ab

b x

(a) Todos os centros estão a igual distância das duas retas, portanto estão na bissetriz das retas s e t. Seja A o ponto

de intersecção entre a paralela à reta t passando por C2 e a perpendicular à reta t passando por C1, e seja B o ponto

de intersecção entre a paralela à reta t passando por C3 e a perpendicular à reta t passando por C2. Seja x = r3.

1

Page 38: MA13 { Geometria { AV1 { 2014 - professor.ufabc.edu.brprofessor.ufabc.edu.br/~jair.donadelli/MA13/MA13.pdf · determine a soma dos quadrados das medidas das diagonais AC e BD em fun˘c~ao

Como os triângulos-retângulos AC1C2 e BC2C3 são semelhantes, temos

C1 AC1C2

=C2BC2C3

,

isto é,a − ba + b

=b − xb + x

,

o que implica x = b2

a .

(b) A relação obtida

r3 =r2

2r1

pode ser reformulada comor3

r2=

r2

r1=

ba

,

o que mostra que os três raios formam uma progressão geométrica de razão ba . Como a mesma situação ocorre para

quaisquer três circunferências consecutivas, a sequência r1, r2, . . ., rn, . . . é uma progressão geométrica de razão ba e

termo inicial a. Assim

rn = a ·(

ba

)n−1=

bn−1

an−2 ,

para n = 1, 2, 3, . . ..

2

Page 39: MA13 { Geometria { AV1 { 2014 - professor.ufabc.edu.brprofessor.ufabc.edu.br/~jair.donadelli/MA13/MA13.pdf · determine a soma dos quadrados das medidas das diagonais AC e BD em fun˘c~ao

AV1 - MA 13 - 2011

Questão 2.

Na figura abaixo, a circunferência de centro I é tangente em D ao lado BC do triângulo ABC e é tangente em E e

F aos prolongamentos dos lados AB e AC, respectivamente.

AB

C

D

E

F

I

(1,0) (a) Mostre que AE é igual ao semiperímetro do triângulo ABC.

(1,0) (b) Mostre que o ângulo AIB é a metade do ângulo ACB.

UMA SOLUÇÃO

(a) Seja 2p o perímetro do triângulo ABC. Tem-se

2p = AB + BC + CA = AB + BD + DC + CA = AB + BE + CF + CA = AE + AF = 2AE .

Logo AE = p.

(b) No triângulo ABC, sejam BAC = A e ACB = C. O ângulo externo de vértice B é DBE = A + C. Seja AIB = θ.

Como AI e BI são bissetrizes dos ângulos CAB e DBE então, no triângulo ABI, o ângulo externo IBE é tal que

A + C2

=DBE

2= IBE = I AB + AIB =

A2+ θ .

Logo

θ =C2

.

3

Page 40: MA13 { Geometria { AV1 { 2014 - professor.ufabc.edu.brprofessor.ufabc.edu.br/~jair.donadelli/MA13/MA13.pdf · determine a soma dos quadrados das medidas das diagonais AC e BD em fun˘c~ao

AV1 - MA 13 - 2011

Questão 3.

(2,0) Dado um paralelogramo ABCD construa no seu exterior os triângulos equiláteros BCE e CDF. Mostre que o

triângulo AEF é equilátero.

UMA SOLUÇÃO

A

BC

D

E

F

α

Primeiro, vemos que BA = DF = CF. A segunda igualdade é consequência de CDF ser equilátero, enquanto a

primeira segue de que AB = CD (pois ABCD é paralelogramo) e CD = DF (pois CDF é equilátero).

Depois, vemos que AD = BE = EC. A segunda desigualdade segue de BCE ser equilátero. A primeira segue de

que AD = BC (pois ABDC é paralelogramo) e BC = BE (pois BCE é equilátero).

Finalmente, vamos mostrar que os ângulos ABE, ECF e ADF são iguais. Para isso vamos mostrar que todos

são iguais a α + 60o, em que α é o ângulo ABC. De fato, isso é evidente para ABE, pois BCE equilátero implica

CBE = 60o. O mesmo para ADF, pois ADC = α (ângulos opostos do paralelogramos são iguais) e CDF = 60o

(CDF é equilátero). Finalmente, em torno do ponto C tem-se

BCD + DCF + FCE + ECE = 360o ,

logo

(180o − α) + 60o + ECF + 60o = 360o

e, portanto, ECF = α + 60o, como queríamos demonstrar.

Portanto os triângulos ABE, FCE e FDA são congruentes, de onde concluímos que AE = EF = AF, isto é, AEF é

equilátero.

4

Page 41: MA13 { Geometria { AV1 { 2014 - professor.ufabc.edu.brprofessor.ufabc.edu.br/~jair.donadelli/MA13/MA13.pdf · determine a soma dos quadrados das medidas das diagonais AC e BD em fun˘c~ao

AV1 - MA 13 - 2011

Questão 4.

(2,0) No triângulo ABC, B = 68o e C = 40o, AD e BE são alturas, M é médio de BC e N é médio de AC. Calcule os

ângulos DNM e EDN.

UMA SOLUÇÃO

A

B CD

E

N

M

(A figura não foi desenhada com os ângulos prescritos no enunciado)

(a) Primeiro, BAC = 180o − 68o − 40o = 72o. Segundo, como N é o ponto médio de AC, então é equidistante de

A e D. Logo AND é isósceles e ND = NA. Pela mesma razão NA = NC, de onde resulta que NDC é isósceles.

Disso resulta que NDC = ACB = 40o e que DNC = 180o − 40o − 40o = 100o. Terceiro, MN é paralelo a BA, logo

MNC é semelhante a BAC e, por conseguinte, MNC é igual a BAC, isto é, 72o. Portanto, DNM = DNC − MNC =

100o − 72o = 28o.

(b) ADN é isósceles e AND = 180o − DNC = 80o, logo ADN = 50o.

Como BDA = 90o = BEA, então E e D pertencem à circunferência cujo diâmetro é AB. Logo, os ângulos ABE e

ADE inscritos nessa circunferência são iguais. Então ADE = ABE = 90o − 72o = 18o.

Portanto EDN = ADN − ADE = 50o − 18o = 32o.

5

Page 42: MA13 { Geometria { AV1 { 2014 - professor.ufabc.edu.brprofessor.ufabc.edu.br/~jair.donadelli/MA13/MA13.pdf · determine a soma dos quadrados das medidas das diagonais AC e BD em fun˘c~ao

AV1 - MA 13 - 2011

Questão 5.

(2,0) O triângulo equilátero ABC está inscrito em uma circunferência e P é um ponto qualquer do menor arco BC.

Prove que PA = PB+ PC (isto é, que a distância de P ao ponto A é igual à soma das distâncias de P aos pontos

B e C).

Sugestão: Considere um ponto D sobre PA tal que PD = PB.

UMA SOLUÇÃO

A

B C

D

P

Seja D o ponto do segmento PA tal que PD = PB. Precisamos mostrar que AD = PC.

Como o arco AB mede 120o, então BPA = 60o. Então BPD = 60o (é o mesmo ângulo) e, como PB = PD, então

PBD é equilátero, resultando que BD = PB. Também por PBD ser equilátero tem-se BDP = 60o e, por conseguinte,

BDA = 120o.

Como o arco BAC mede 240o, então BPC = 240o2 = 120o, logo BPC = BDA. Juntando essa informação com a

igualdade BAP = BCP, que é evidente da simetria da construção, concluímos que ABD = PBC.

Por LAL os triângulos ABD e CBP são congruentes, resultando que AD = PC, como queríamos demonstrar.

6

Page 43: MA13 { Geometria { AV1 { 2014 - professor.ufabc.edu.brprofessor.ufabc.edu.br/~jair.donadelli/MA13/MA13.pdf · determine a soma dos quadrados das medidas das diagonais AC e BD em fun˘c~ao

MA13 – Geometria I – Avaliação 2 – 2011

Gabarito

Questão 1

(2,0) A figura abaixo mostra um triângulo equilátero e suas circunferências inscrita e

circunscrita. A circunferência menor tem raio 1.

Calcule a área da região sombreada.

Uma solução:

Seja O, o centro do triângulo equilátero ABC e seja M o ponto médio do lado BC como na

figura acima. Pela propriedade do baricentro do triângulo, OMOA 2 e como 1OM temos

2OA .

A região cuja área se pede é formada por duas partes justapostas X e Y como mostra a figura.

Observando que YX 33 é a área da coroa circular formada pelas duas circunferências temos

312)(3 22YX .

Logo, YX .

A

B C M

O

X

Y

Page 44: MA13 { Geometria { AV1 { 2014 - professor.ufabc.edu.brprofessor.ufabc.edu.br/~jair.donadelli/MA13/MA13.pdf · determine a soma dos quadrados das medidas das diagonais AC e BD em fun˘c~ao

Questão 2

O poliedro P que inspirou a bola da Copa de 70 é formado por faces

pentagonais e hexagonais, e é construído da seguinte forma:

•Considere um icosaedro regular de aresta a (Fig. 1 abaixo).

•A partir de um vértice e sobre cada uma das 5 arestas que concorrem nesse

vértice, assinale os pontos que estão a uma distância de 3

a desse vértice. Esses

5 pontos formam um pentágono regular (Fig. 2).

•Retirando a pirâmide de base pentagonal que ficou formada obtemos a Fig. 3.

•Repetindo a mesma operação para todos os vértices do icosaedro obtém-se o poliedro P.

(0,5) (a) Determine quantas são as faces pentagonais e quantas são as faces hexagonais de P.

(0,7) (b) Determine os números de arestas, faces e vértices de P.

(0,8) (c) Sabendo que uma diagonal de um poliedro é todo segmento que une dois vértices que

não estão na mesma face, determine o número de diagonais de P.

Uma solução:

(a) Cada face pentagonal de P apareceu onde havia um vértice do icosaedro. Como o icosaedro

tem 12 vértices então P tem 12 faces pentagonais. Cada face (triangular) do icosaedro deu

origem a uma face hexagonal de P. Como o icosaedro tem 20 faces triangulares então P tem 20

faces hexagonais.

(b) Do item anterior temos 125F e 206F

O número total de faces de P é 32201265 FFF .

Contando as arestas temos: 180206125652 65 FFA , ou seja, 90A .

Como P é convexo então vale a relação de Euler 2FAV . Portanto, 60V .

(c) Seja nd o número de diagonais de um polígono de n lados.

O número de diagonais de um pentágono é 55d e o de um hexágono é 96d .

Fig. 1 Fig. 2 Fig. 3

Page 45: MA13 { Geometria { AV1 { 2014 - professor.ufabc.edu.brprofessor.ufabc.edu.br/~jair.donadelli/MA13/MA13.pdf · determine a soma dos quadrados das medidas das diagonais AC e BD em fun˘c~ao

A soma dos números de diagonais de todas as faces é 240205126655 dFdFS .

Vamos agora construir todos os segmentos cujas extremidades são os V vértices do poliedro P.

A quantidade de diagonais de P é SACD V

2 .

Assim, 14403301170240902

5960240902

60CD .

Page 46: MA13 { Geometria { AV1 { 2014 - professor.ufabc.edu.brprofessor.ufabc.edu.br/~jair.donadelli/MA13/MA13.pdf · determine a soma dos quadrados das medidas das diagonais AC e BD em fun˘c~ao

Questão 3

Definição: Dado um segmento AB, o plano mediador desse segmento é o plano perpendicular a

AB que contém o seu ponto médio.

1ª Parte

(2,0) Prove que um ponto P equidista de dois pontos A e B se, e somente se, pertence ao plano

mediador de AB.

Uma solução:

Seja M o ponto médio de AB e seja Π o plano mediador de AB.

(a) Suponha que P pertença a Π. Se P coincide com M então

equidista de A e B. Se não, como AB é perpendicular a Π então AB

é perpendicular a MP. Como M é médio de AB então os triângulos

retângulos MPA e MPB são congruentes.

Logo, PBPA , ou seja, P equidista de A e B.

(b) Suponha que P não pertença a Π. Imaginemos, por exemplo e

sem perda de generalidade, os pontos P e A no mesmo semiespaço

determinado por Π. Como B está no semiespaço oposto a reta PQ

corta Π em um ponto Q. Como Q então, pela parte a),

QBQA .

No triângulo PAQ tem-se: PBQBPQQAPQPA .

Assim, P não equidista de A e B.

P

A

B

M

Π

P

A

B

Q

Π

Page 47: MA13 { Geometria { AV1 { 2014 - professor.ufabc.edu.brprofessor.ufabc.edu.br/~jair.donadelli/MA13/MA13.pdf · determine a soma dos quadrados das medidas das diagonais AC e BD em fun˘c~ao

2ª Parte

A figura abaixo mostra o cubo ABCD-EFGH de aresta a.

Sejam M, N, P, Q, R e S os pontos médios das arestas

AB, BF, FG, GH, HD e DA.

(0,5) (a) Mostre que esses seis pontos são coplanares.

Sugestão: Mostre que qualquer um deles pertence ao plano

mediador da diagonal EC do cubo (a propriedade enunciada na

primeira parte da questão pode ser utilizada mesmo que você não a

tenha demonstrado).

(0,5) (b) Mostre que o hexágono MNPQRS é regular.

(1,0) (c) Calcule o volume da pirâmide de vértice E e base MNPQRS.

Uma solução:

(a) Tomemos o ponto M, médio da aresta AB. Os

triângulos AME e BMC são congruentes, pois BMAM ,

BCAE e 090MBCMAE

Logo, MCME e, portanto, M pertence ao plano

mediador da diagonal EC.

Analogamente, cada um dos outros pontos: N, P, Q, R e S

também estão nesse mesmo plano.

(b) Cada lado do hexágono é a metade da diagonal de

uma face. Por exemplo, 2

2

2

aBGNP .

Seja O, o centro do cubo. Todos os vértices do

hexágono possuem mesma distância ao ponto O. A

distância do centro do cubo a qualquer aresta é a

metade da diagonal de uma face, ou seja, 2

2a.

Portanto, cada um dos triângulos MON, NOP, ...,

SOM é equilátero e o hexágono é regular.

(c) A área do hexágono é 2

33

4

36

22 aa.

Como a altura da pirâmide é a metade da diagonal do cubo temos 2

3aOE .

O volume da pirâmide é: 8

3

2

3

2

33

3

1 32 aaaV .

A B

C

D

E

F

G

H

A

C

E

M B

M

G

H

A

B

C

E

F

N

P

Q

R

S

O

Page 48: MA13 { Geometria { AV1 { 2014 - professor.ufabc.edu.brprofessor.ufabc.edu.br/~jair.donadelli/MA13/MA13.pdf · determine a soma dos quadrados das medidas das diagonais AC e BD em fun˘c~ao

3ª Parte

A figura abaixo mostra o cubo ABCD-EFGH de aresta a.

(1,0) (a) Mostre que as retas DB e EC são ortogonais.

(1,0) (b) Calcule o comprimento da perpendicular

comum entre DB e EC.

Uma solução:

(a) Seja Π o plano diagonal AEGC.

Como AE é perpendicular ao plano ABCD então AE é ortogonal a BD. Mas AC é perpendicular

a BD (pois as diagonais de um quadrado são perpendiculares. Como BD é ortogonal a AE e AC

então BD é perpendicular a Π.

Como EC está contida em Π então BD é ortogonal a EC.

(b) Seja X o ponto onde BD fura o plano Π. O ponto X é o centro da face ABCD.

Sobre o plano Π tracemos XY perpendicular a EC.

Lembrando que BD é perpendicular a Π então BD é perpendicular a XY. Assim, XY é a

perpendicular comum entre BD e EC.

Os triângulos retângulos CYX e CAE são semelhantes. Logo,

CE

CX

AE

XY →

3

22

a

a

a

XY →

6

6aXY

A B

C

D

E

F

G H

A

B

C

D

E

X

G

H

Y

Π

Page 49: MA13 { Geometria { AV1 { 2014 - professor.ufabc.edu.brprofessor.ufabc.edu.br/~jair.donadelli/MA13/MA13.pdf · determine a soma dos quadrados das medidas das diagonais AC e BD em fun˘c~ao

MA13 – Geometria I – Avaliação 3 – 2011

Questão 1

Considere um quadrado ABCD de lado a e seja E o ponto do lado CD tal que

AE BC CE .

(1,0) (a) Calcule o comprimento de CE.

(1,0) (b) Calcule o seno do ângulo C ˆ A E .

Questão 2

Um trapézio ABCD tem altura h e bases AB a e CD b. Seja F o ponto de

interseção das diagonais.

(1,0) (a) Calcule as distâncias de F às duas bases.

(1,0) (b) Calcule as áreas dos triângulos ADF e BCF.

Questão 3

Seja ABC um triângulo qualquer. Desenhe exteriormente a ABC os triângulos

equiláteros ABD e ACE.

(1,0) (a) Mostre que DC = BE. Sugestão: use congruência de triângulos.

(0,5) (b) Sendo F o ponto de interseção de DC e BE, mostre que o quadrilátero ADBF

é inscritível.

(0,5) (c) Mostre que A ˆ F B B ˆ F C C ˆ F A 1200.

Questão 4

Seja um plano horizontal. A reta r é perpendicular a e seja A o ponto de

interseção de r e . A reta s está contida em e não passa por A. O ponto B da reta s

é tal que AB é perpendicular à reta s. Seja M um ponto de r e N um ponto de s.

Dados: AM a, BN b, AB c .

(0,5) (a) Faça um desenho da situação descrita no enunciado.

(0,5) (b) Calcule a distância entre os pontos M e N.

(0,5) (c) Calcule a tangente do ângulo que a reta MN faz com o plano .

(0,5) (d) Calcule a tangente do ângulo entre as retas AB e MN.

Questão 5

As bases de um tronco de pirâmide regular são quadrados de lados 12 e 4. Sabe-se

que a área lateral é igual à soma das áreas das bases.

(1,0) (a) Calcule a altura do tronco.

(1,0) (b) Calcule o volume do tronco.

Page 50: MA13 { Geometria { AV1 { 2014 - professor.ufabc.edu.brprofessor.ufabc.edu.br/~jair.donadelli/MA13/MA13.pdf · determine a soma dos quadrados das medidas das diagonais AC e BD em fun˘c~ao

MA13 – Geometria I – Avaliação 3 – 2011 Gabarito

Questão 1 – Solução

(a) Seja CE x . Assim AE a x .

Traçando EF perpendicular a AB temos no triângulo AEF:

(a x)2 (a x)2 a2 o que dá xa

4.

(b) Seja A ˆ E C .

Como CEa

4 e AE a

a

4

5a

4 temos, pela lei dos senos,

22

45

sin

4 aa o que dá sin

2

10.

Questão 2 – Solução

(a) Sejam x e y as distâncias de F às bases AB e

CD, respectivamente. Como os triângulos FAB e

FCD são semelhantes, temos: a

x

b

y

a b

h

Assim, xah

a b e y

bh

a b.

(b) Os triângulos ADB e ACB têm mesma área porque possuem mesma base e mesma

altura. Os triângulos ADF e BCF têm mesma área porque

[ADF] = [ADB] – [AFB] = [ACB] – [AFB] = [BCF]

)(2222][][

ba

abh

ba

ahh

aaxahBCFADF

A B

C D

F

x

h

y

b

a

A B

C D E

F

a

Page 51: MA13 { Geometria { AV1 { 2014 - professor.ufabc.edu.brprofessor.ufabc.edu.br/~jair.donadelli/MA13/MA13.pdf · determine a soma dos quadrados das medidas das diagonais AC e BD em fun˘c~ao

Questão 3 – Solução

(a) Temos AD AB , AC AE e D ˆ A C B ˆ A E ˆ A 600. Portanto, os triângulos

ADC e ABE são congruentes e DC = BE.

(b) Pela congruência anterior, A ˆ D F A ˆ B F . Portanto D está na circunferência que

passa por A, B e F.

(c) Como ADBF é inscritível, seus ângulos são suplementares. Então

A ˆ F B 1800 A ˆ D B 1800 600 1200. Analogamente, AECF é inscritível e

C ˆ F A 1200. Consequentemente, 0120ˆCFB .

Questão 4 – Solução

(a)

b) No triângulo ABN, retângulo em B, AN 2 b2 c 2.

No triângulo MAN, retângulo em A, MN 2 a2 AN 2 a2 b2 c 2.

Então 222 cbaMN .

c) O ângulo que MN faz com é M ˆ N A . Assim, 22

tancb

a

AN

AM.

d) Construa o retângulo ABNP.

AM é ortogonal a NP e AP é perpendicular a NP. Portanto, NP é perpendicular ao

plano AMP e, consequentemente, o ângulo NPA é reto.

O ângulo entre MN e BA é o ângulo entre MN e NP, M ˆ N P .

Assim, c

ba

NP

PM 22

tan .

A

B

M

N

P

r

s

a

b

c

Page 52: MA13 { Geometria { AV1 { 2014 - professor.ufabc.edu.brprofessor.ufabc.edu.br/~jair.donadelli/MA13/MA13.pdf · determine a soma dos quadrados das medidas das diagonais AC e BD em fun˘c~ao

Questão 5 – Solução

(a)

Sejam O e O os centros

das duas bases (maior e

menor) como mostra a

figura acima.

Na reta OO está o

vértice V da pirâmide que

dou origem ao tronco.

A altura do tronco é

hOO .

Cada face lateral do

tronco é um trapézio

isósceles, e a altura de

um dos trapézios é o segmento MN que une os pontos médios das duas bases. Seja

xMN .

A área lateral do tronco é a soma das áreas dos quatro trapézios. Então,

22 4122

)412(4

x

Isto dá 5x . Trace agora NP perpendicular à OM como na figura acima. Temos

hNPOO , 2OPON , 6OM e, consequentemente, 4PN . No triângulo

PMN retângulo em P temos 3h .

(b) Seja yOV .

Utilizando a semelhança entre as duas pirâmides temos 12

4

3y

y o que dá

2

3y .

A altura da pirâmide grande é 2

9

2

33OV e o seu volume é

2162

912

3

1 2

1V .

O volume da pirâmide pequena é 82

34

3

1 2

2V .

O volume do tronco é a diferença: 2088216V unidades de volume.

Obs:

Pode-se também aplicar a fórmula do volume do tronco de pirâmide:

)(3

2121 SSSSh

V onde 1S e 2S são as áreas das duas bases e h é a altura do

tronco. Assim,

2084816144)412412(3

3 2222V .

O

O'

M

N

P

12

V

4

x h

Page 53: MA13 { Geometria { AV1 { 2014 - professor.ufabc.edu.brprofessor.ufabc.edu.br/~jair.donadelli/MA13/MA13.pdf · determine a soma dos quadrados das medidas das diagonais AC e BD em fun˘c~ao

MA13 Geometria I Avaliacao 1 2012

SOLUCOES

Questao 1. (pontuacao: 2)

O ponto D pertence ao lado AC do triangulo ABC. Sabe-se que AB = BC = CD e que o angulo ABD mede 21o.

Determine a medida do angulo ABC.

A

B

C D

21o

α α

Uma solucao:

Como AB = BC, seja α = ∠BAC = ∠BCA. O angulo BDC e externo do triangulo ABD. Entao, ∠BDC =

21o + α = ∠DBC, pois BC = CD. No triangulo BDC temos 21o + α + 21o + α + α = 180o, ou seja, α = 46o. O

angulo ABC mede 21o + 21o + α = 42o + 46o = 88o .

Questao 2. (pontuacao: 2)

Quadrados foram construıdos sobre os lados de um paralelogramo como mostra a figura abaixo. Mostre que os

centros desses quatro quadrados sao vertices de outro quadrado.

A

C

E

F

G

H

α

β D

B

x

x y

Page 54: MA13 { Geometria { AV1 { 2014 - professor.ufabc.edu.brprofessor.ufabc.edu.br/~jair.donadelli/MA13/MA13.pdf · determine a soma dos quadrados das medidas das diagonais AC e BD em fun˘c~ao

Uma solucao:

No paralelogramo ABCD os quadrados construıdos sobre os lados AB, BC, CD e DA tem centros E, F , G e H,

respectivamente.

Os triangulos AEB, BFC, CGD e DHA sao retangulos e isosceles. O primeiro e o terceiro sao congruentes e o

segundo e o quarto sao tambem congruentes.

Sejam ∠BAD = α e ∠ADC = β dois angulos internos vizinhos do paralelogramo. Sabemos que α+ β = 180o.

Observemos que ∠HAE = 45o + α+ 45o = 90o + α e que ∠HDG = 360o − 45o − 45o − β = 270o − (180o − α) =

90o + α = ∠HAE.

Reunindo as informacoes anteriores concluımos que os triangulos HAE, HDG, FCG e FBE sao todos congruentes

e, portanto, EH = HG = GF = FE e o quadrilatero EFGH possui os quatro lados iguais.

Da congruencia dos triangulos HAE e HDG temos ∠AHE = ∠DHG = x e seja ∠EHD = y. Por um lado,

∠AHE + ∠EHD = x+ y = 90o, pois o angulo AHD e reto.

Por outro lado, ∠EHG = ∠DHG+ ∠EHD = x+ y = 90o .

Assim, o quadrilatero EFGH possui os quatro lados iguais e um angulo reto. Logo, e um quadrado.

Questao 3. (pontuacao: 2)

No triangulo ABC de lados AB = 8, BC = 7 e AC = 9, os pontos M e N dos lados AB e AC, respectivamente,

sao tais que o segmento MN e tangente a circunferencia inscrita no triangulo ABC. Mostre que o perımetro do

triangulo AMN e constante e calcule seu valor.

Uma solucao:

A

B C

N

M

P

Q

R

x y

z

Sejam AM = x, MN = y e NA = z os lados do triangulo AMN . Temos MB = 8 − x e CN = 9 − z . Como o

quadrilatero BCNM e circunscritıvel temos, pelo teorema de Pitot (Unidade 7, Teorema 4), BC+MN = MB+NC

Page 55: MA13 { Geometria { AV1 { 2014 - professor.ufabc.edu.brprofessor.ufabc.edu.br/~jair.donadelli/MA13/MA13.pdf · determine a soma dos quadrados das medidas das diagonais AC e BD em fun˘c~ao

ou seja, 7+y = 8−x+9−z. Logo x+y+z = 10. Portanto o perımetro do triangulo AMN e igual a 10, independente

da posicao do segmento MN .

Outra solucao:

A circunferencia inscrita em ABC e uma circunferencia exiscrita ao triangulo AMN . Sabemos que o semiperımetro

do triangulo AMN e o segmento AP que e constante, ou seja, nao depende da posicao do segmento MN (Unidade

7, Proposicao 22). Fazendo AP = AR = a, BP = BQ = b e CQ = CR = c, temos as equacoes:a+ b = 8

b+ c = 7

c+ a = 9

Resolvendo, encontramos a = 5 que e o semiperımetro do triangulo AMN . Logo, o perımetro de AMN e 10.

Questao 4. (pontuacao: 2)

No trapezio ABCD os angulos A e D sao retos, AB = 12, CD = 4 e AD = 10. O ponto E pertence ao lado AD e o

ponto F pertence ao lado BC. Sabe-se que as retas EF e AB sao paralelas e que o segmento EF fica dividido em

tres partes iguais pelas diagonais do trapezio. Calcule a distancia entre as retas AB e EF .

Uma solucao:

O problema tem duas solucoes pois ha duas possibilidades: quando EF esta abaixo do encontro das diagonais do

trapezio e quando EF esta acima do encontro das diagonais do trapezio. Qualquer uma das solucoes esta igualmente

correta.

Primeira situacao:

A B

C D

E F P Q

x

m m m

10 – x

10

12

4

Page 56: MA13 { Geometria { AV1 { 2014 - professor.ufabc.edu.brprofessor.ufabc.edu.br/~jair.donadelli/MA13/MA13.pdf · determine a soma dos quadrados das medidas das diagonais AC e BD em fun˘c~ao

Na figura acima, seja AE = x. Entao, ED = 10− x. Como as diagonais dividem EF em tres partes iguais sejam

EP = PQ = QF = m. Da semelhanca dos triangulos AEP e ADC temos: m4 = x

10 ⇒ m = 2x5 . Da semelhanca

dos triangulos DEQ e DAB temos: 2m12 = 10−x

10 ⇒ m = 3(10−x)5 .

Igualando temos 2x = 3(10− x), o que da x = 6.

Segunda situacao:

A B

C D

E F P Q

x

m m m

10

12

4

Na figura acima, seja AE = x. Entao, ED = 10− x. Como as diagonais dividem EF em tres partes iguais sejam

EP = PQ = QF = m. Da semelhanca dos triangulos QEA e CDA temos: 2mx = 4

10 ⇒ m = x5 . Da semelhanca

dos triangulos DEP e DAB temos: 10−xm = 10

12 ⇒ m = 12(10−x)10 .

Igualando obtemos x = 607 .

Questao 5. (pontuacao: 2)

A figura abaixo mostra o triangulo acutangulo ABC inscrito na circunferencia de centroO. A retaBD e perpendicular

em D a AC e encontra a circunferencia em M . A reta CE e perpendicular em E a AB e encontra a circunferencia

em N . As alturas BD e CE intersectam-se em H, ortocentro do triangulo.

a) Mostre que HD = DM .

b) Mostre que MN e perpendicular a OA.

Page 57: MA13 { Geometria { AV1 { 2014 - professor.ufabc.edu.brprofessor.ufabc.edu.br/~jair.donadelli/MA13/MA13.pdf · determine a soma dos quadrados das medidas das diagonais AC e BD em fun˘c~ao

Uma solucao:

A

B C

D

M

E N O H

α

β β

α α

a) Considerando a figura acima, sejam ∠DCH = α e ∠DHC = β. Como o angulo HDC e reto entao α e β

sao complementares. Temos ∠EHB = β (oposto pelo vertice de DHC) e ∠HBE = α pois o angulo BEH e reto.

Escrevemos ∠ABM = ∠HBE = α. Como os angulos inscritos ABM e ACM subtendem o mesmo arco AM , entao

sao iguais, ou seja, ∠ACM = α.

Os triangulos retangulos CDH e CDM sao congruentes. Assim HD = DM , como querıamos demonstrar.

b)

A M

N O

C α α

Os arcos AM e AN sao iguais porque ∠ACM = ∠ACN = α. Como arcos iguais subtendem cordas iguais o ponto

A equidista dos pontos M e N . Entretanto o ponto O, centro da circunferencia tambem equidista de M e N . Assim,

A e O sao pontos da mediatriz do segmento MN o que significa dizer que a reta AO e a mediatriz do segmento MN .

Logo, OA e perpendicular a MN .

Page 58: MA13 { Geometria { AV1 { 2014 - professor.ufabc.edu.brprofessor.ufabc.edu.br/~jair.donadelli/MA13/MA13.pdf · determine a soma dos quadrados das medidas das diagonais AC e BD em fun˘c~ao

MESTRADO PROFISSIONAL EM MATEMATICA EM REDE NACIONAL

Avaliacao 2 - MA13 - 2015 - Gabarito

Questao 01 [ 2,00 pts ]

Em um ponto da borda de um pasto circular, amarra-se um bode, com uma corda cujo comprimento e√

3 vezes

o raio do pasto. O bode pode comer todo o pasto que conseguir alcancar, podendo deslocar-se ate a corda estar

totalmente esticada. Determine a fracao maxima do pasto que o bode pode comer.

Solucao

Sejam O o centro do pasto, R a medida de seu raio e P o ponto onde a corda, de comprimento R√

3, esta presa.

A area S do pasto que o bode pode comer pode ser decomposto em uma area S1 mais duas areas identicas S2 e S3, conforme

a figura abaixo.

Como OP = OA = R e AP = R√

3, temos

(R√

3)2

= R2 +R2 − 2 ·R ·R · cos(POA

)

logo

cos(POA

)= −1

2,

o que nos da POA = 120◦. Com isso, como o triangulo POA e isosceles de vertice O, temos OPA = 12(180◦ − 120◦) = 30◦.

Teremos entao APB = 60◦, logo a area do setor circular S1, de raio R√

3 e angulo APB = 60◦ e dada por

Page 59: MA13 { Geometria { AV1 { 2014 - professor.ufabc.edu.brprofessor.ufabc.edu.br/~jair.donadelli/MA13/MA13.pdf · determine a soma dos quadrados das medidas das diagonais AC e BD em fun˘c~ao

S1 =60◦

360◦ · π(R√

3)2

=3πR2

6=πR2

2.

A area S2, por sua vez, corresponde a area do setor circular de raio R e angulo 120◦, subtraindo a area do triangulo AOP .

Assim,

S2 =120◦

360◦ · πR2 − R ·R · sen 120◦

2=πR2

3− R2

√3

4.

Como S3 = S2, temos

S = S1 + S2 + S3 = S1 + 2S2 =πR2

2+ 2

(πR2

3− R2

√3

4

)logo

S =7π − 3

√3

6·R2.

Questao 02 [ 2,00 pts ]

A figura mostra uma plataforma de base quadrada ABCD e uma torre central em forma de piramide quadrangular

de base PQRS e vertice H, cujo volume e 576 m3. As diagonais PR e QS estao contidas, respectivamente, nas

diagonais AC e BD. As arestas das bases da plataforma e da piramide medem, respectivamente, 19√

2 m e 6√

2 m.

Para aumentar a seguranca, quatro cabos de aco serao presos e esticados entre cada ponto medio das arestas laterais

da piramide e o vertice mais proximo da base da plataforma.

(a) Sendo M o ponto medio da aresta HQ e N a projecao de M na base da piramide, determine a medida do

segmento MN .

(b) Determine o comprimento do cabo de aco BM .

Solucao

(a) Sendo V o volume da piramide e O o centro do quadrado PQRS, temos:

V =1

3· PQ2 ·OH ∴ 576 =

1

3·(

6√

2)2

·OH ∴ OH = 24m.

Como M e ponto medio de QH, entao MN e base media do triangulo OQH e, por isso:

MN =OH

2∴MN = 12m.

(b) Aplicando o Teorema de Pitagoras no triangulo PQS, retangulo em P , temos:

QS2

= PQ2

+ PS2

=(

6√

2)2

+(

6√

2)2

∴ QS = 12 ∴ OQ =QS

2= 6m.

Page 60: MA13 { Geometria { AV1 { 2014 - professor.ufabc.edu.brprofessor.ufabc.edu.br/~jair.donadelli/MA13/MA13.pdf · determine a soma dos quadrados das medidas das diagonais AC e BD em fun˘c~ao

Como o segmento MN e base media do triangulo OQH, entao N e ponto medio de OQ, sendo:

ON =OQ

2= 3m.

Aplicando o Teorema de Pitagoras no triangulo ABD, retangulo em A, temos:

BD2

= AB2

+AD2

=(

19√

2)2

+(

19√

2)2

∴ BD = 38 ∴ OB =BD

2= 19m.

Assim segue que:

BN = OB −ON = 19− 3 ∴ BN = 16m.

Aplicando o Teorema de Pitagoras no triangulo MNB, retangulo em N , temos:

BM2

= MN2

+BN2

= 122 + 162 ∴ BM = 20m.

Questao 03 [ 2,00 pts ]

(a) Prove que todo trapezio inscritıvel e isosceles.

(b) Um trapezio ABCD, de bases AB e CD, e inscritıvel e circunscritıvel. Sabendo que AB > CD, prove que

AB > BC.

Solucao

(a) Seja ABCD um trapezio inscritıvel, com bases AB e CD.

Como o trapezio e inscritıvel, temos que A+ C = 180◦. Porem, como AB e CD sao paralelos, temos B + C = 180◦. As duas

igualdades nos dao, juntas, que A = B, o que implica que o trapezio e isosceles.

(b) Como ABCD e inscritıvel, pelo item (a) ele sera isosceles, logo BC = AD.

Por outro lado, o trapezio e circunscritıvel, logo

AB + CD = BC +AD.

Mas, como BC = AD, temos

AB + CD = 2BC.

E, como AB > CD, temos

2AB = AB +AB > AB + CD = 2BC,

logo

AB > BC.

Questao 04 [ 2,00 pts ]

Seja ABCD um losango de diagonais de medidas AC = 2a e BD = 2b. Dos pontos A e C, tracamos os segmentos

AE e CF , de medidas AE = x e CF = y, perpendiculares ao plano que contem o losango e de um mesmo lado deste

plano. Calcule o volume do solido ABCDEF .

Page 61: MA13 { Geometria { AV1 { 2014 - professor.ufabc.edu.brprofessor.ufabc.edu.br/~jair.donadelli/MA13/MA13.pdf · determine a soma dos quadrados das medidas das diagonais AC e BD em fun˘c~ao

Solucao

Como pode ser visto na figura, o solido ABCDEF e formado pela uniao de duas piramides:

• a piramide cuja base e o trapezio ACFE e cuja altura e PB e

• a piramide cuja base e o trapezio ACFE e cuja altura e PD,

sendo P o ponto de encontro das diagonais do losango ABCD.

Como PB = PD = b, o volume de cada uma dessas piramides e dado por

V1 =1

3· AreaACFE · b =

1

3·(x+ y

2· 2a

)· b =

ab(x+ y)

3.

Assim, o volume do solido ABCDEF e

VABCDEF = 2 · V1 =2ab(x+ y)

3.

Questao 05 [ 2,00 pts ]

(a) Usando apenas a relacao fundamental da trigonometria e as formulas de adicao de arcos e de arcos duplos prove

que:

cos(3x) = 4 cos3(x)− 3 cos(x).

(b) Resolva a equacao 4 cos3(x)− 3 cos(x) = cos(π

5

).

Solucao

Page 62: MA13 { Geometria { AV1 { 2014 - professor.ufabc.edu.brprofessor.ufabc.edu.br/~jair.donadelli/MA13/MA13.pdf · determine a soma dos quadrados das medidas das diagonais AC e BD em fun˘c~ao

(a) Aplicando a formula de adicao de arcos temos:

cos(3x) = cos(2x+ x) = cos(2x) cos(x)− sen(2x)sen(x).

Pelas formulas de arcos duplos segue que:

cos(3x) =[cos2(x)− sen2(x)

]cos(x)− [2sen(x) cos(x)] sen(x)

= cos3(x)− 3sen2(x) cos(x)

= cos3(x)− 3[1− cos2(x)

]cos(x)

= 4 cos3(x)− 3 cos(x).

(b) Por (a) segue que 4 cos3(x)− 3 cos(x) = cos(3x).

Logo cos(3x) = cos(π

5

).

Portanto 3x = ±π5

+ 2kπ, k ∈ Z, ou seja, x = ± π

15+

2

3kπ, k ∈ Z.

Page 63: MA13 { Geometria { AV1 { 2014 - professor.ufabc.edu.brprofessor.ufabc.edu.br/~jair.donadelli/MA13/MA13.pdf · determine a soma dos quadrados das medidas das diagonais AC e BD em fun˘c~ao

MESTRADO PROFISSIONAL EM MATEMATICA EM REDE NACIONAL

Avaliacao 1 - MA13 - 2015 - Gabarito

Questao 01 [ 2,00 pts ]

Em um quadrilatero convexo ABCD, prove que

MN 6AB + CD

2,

onde M e N sao os pontos medios dos lados BC e AD, respectivamente.

Sugestao: Utilize o ponto medio da diagonal AC.

Solucao

Tomando P como ponto medio de AC, temos que MP =AB

2e PN =

CD

2. Para o ponto P ha duas situacoes possıveis :

(I) M , N e P nao estao alinhados. Da existencia do triangulo MNP , teremos:

MN < MP + PN.

(II) M , N e P estao alinhados, ou seja,

MN = MP + PN.

De (I) e (II) :

MN 6AB + CD

2.

Page 64: MA13 { Geometria { AV1 { 2014 - professor.ufabc.edu.brprofessor.ufabc.edu.br/~jair.donadelli/MA13/MA13.pdf · determine a soma dos quadrados das medidas das diagonais AC e BD em fun˘c~ao

Questao 02 [ 2,00 pts ]

Na figura, M e ponto medio de AB.

Descreva a construcao com regua e compasso da circunferencia C, tangente a reta r e ao segmento AB, e tal que M

seja o ponto de tangencia de C com AB.

Solucao

Vamos imaginar a circunferencia construıda, para buscar um ponto chave da construcao.

Como a circunferencia e tangente ao segmento AB em seu ponto medio M , o centro O estara sobre a mediatriz m de AB. E,

como a circunferencia e tangente tanto a↔AB quanto a reta r, seu centro estara sobre a bissetriz b entre r e

↔AB.

Assim, basta construir m e b.

Para construir m, tomamos o compasso e, com uma mesma abertura maior que 12AB, tracamos duas circunferencias, uma

de centro A e outra de centro B. Estas duas circunferencias irao se intersectar em dois pontos, X e Y . Construımos entao

m =↔XY .

Page 65: MA13 { Geometria { AV1 { 2014 - professor.ufabc.edu.brprofessor.ufabc.edu.br/~jair.donadelli/MA13/MA13.pdf · determine a soma dos quadrados das medidas das diagonais AC e BD em fun˘c~ao

Para construir m, prolongamos AB ate C ∈↔AB ∩r. Com um raio qualquer, construımos um cırculo de centro C, que

intersectara−→CA em um ponto W . Esta circunferencia intersectara r dois pontos, escolhemos um deles, Z, de forma que

−→CZ

determine o menor angulo ZCA, como na figura. Com abertura maior que 12WZ, construımos duas circunferencias, de centros

W e Z respectivamente, que se intersectam em dois pontos. Sendo T um destes pontos, b =↔CT .

Assim, construıdas as retas m e b, obtemos O ∈ m ∩ b. Para construir a circunferencia pedida, basta fazer O como centro e

tomar OM como abertura do compasso.

Questao 03 [ 2,00 pts ]

Em um triangulo ABC de incentro I, sejam E e F pontos sobre AC tais que−−→BE e bissetriz de ∠ABC e

−−→BF e

bissetriz de ∠EBC. Se ∠BAC mede 60◦ e FI e paralelo a BC, determine as medidas dos angulos ∠ABC e ∠ACB.

Dica: Procure uma relacao entre os triangulos ABI e AFI.

Solucao

Seja ∠BAC = 4θ, teremos entao que ∠ABE = ∠EIF = 2θ e que o triangulo BIF e isosceles.

Sejam M e N os pes das perpendiculares tracadas de I sobre AB e AC, respectivamente.

Da congruencia dos triangulos BIM e NIF , temos que ∠EFI = 2θ e consequentemente no triangulo ABC obtemos 4θ +

2θ + 60◦ = 180◦, ou seja, θ = 20◦.

Logo os angulos pedidos sao 80◦ e 40◦.

Page 66: MA13 { Geometria { AV1 { 2014 - professor.ufabc.edu.brprofessor.ufabc.edu.br/~jair.donadelli/MA13/MA13.pdf · determine a soma dos quadrados das medidas das diagonais AC e BD em fun˘c~ao

Questao 04 [ 2,00 pts ]

Em um trapezio de bases AB e CD, com AB > CD, traca-se por B uma reta paralela a diagonal AC que encontra

o prolongamento de AD em E. Sendo P o ponto de encontro dos prolongamentos dos lados AD e BC, determine

PA em funcao apenas de PD e PE.

Solucao

Sejam DP = a, PE = b, DC = x, AB = y, AC = n e EB = m. Queremos determinar AP = k como funcao de a e b.

Das semelhancas dos triangulos APC e PEB, ADC e ABE, ABP e PDC, temos queb

k=m

n,y

x=m

n,y

x=k

a.

Das igualdades acima encontramos

b

k=k

a, donde k =

√ab.

Page 67: MA13 { Geometria { AV1 { 2014 - professor.ufabc.edu.brprofessor.ufabc.edu.br/~jair.donadelli/MA13/MA13.pdf · determine a soma dos quadrados das medidas das diagonais AC e BD em fun˘c~ao

Questao 05 [ 2,00 pts ]

Seja P um ponto sobre a diagonal AC do quadrado ABCD. Prove que PA2, PB

2, PC

2estao, nesta ordem, em

progressao aritmetica.

Solucao

Sejam Q, R e S os pes das perpendiculares tracadas de P sobre os lados AB, AD e CD, respectivamente. Como P esta sobre

a diagonal AC, temos que PQ = AQ = x e PS = SC = QB = L− x, onde L e a medida do lado do quadrado.

Dos triangulos retangulos PQA, PQB e PSC teremos:

PA2

= x2 + x2 = 2x2,

PB2

= x2 + (L− x)2,

PC2

= 2(L− x)2.

Como

PC2 − PB2

= 2(L− x)2 −(x2 + (L− x)2

)= (L− x)2 − x2,

temos

PB2 − PA2

= x2 + (L− x)2 − 2x2 = (L− x)2 − x2 = PC2 − PB2

.

Com isso, PA2, PB

2e PC

2estao em uma PA.

Page 68: MA13 { Geometria { AV1 { 2014 - professor.ufabc.edu.brprofessor.ufabc.edu.br/~jair.donadelli/MA13/MA13.pdf · determine a soma dos quadrados das medidas das diagonais AC e BD em fun˘c~ao

MESTRADO PROFISSIONAL EM MATEMATICA EM REDE NACIONAL

Avaliacao 2 - MA13 - 2015 - Gabarito

Questao 01 [ 2,00 ]

Considere um cilindro solido de altura 2R, cujas bases sao dois cırculos de raio R, do qual sao retirados dois cones

solidos de alturaR e que tem por base as bases do cilindro, formando-se assim um solido S. Considere ainda uma esfera

de raioR, e que, assim como o solido S, esta sobre um plano.

(a) Prove que, intersectando a esfera e o solido S por um plano paralelo ao plano que apoia estes solidos, como na

figura, obtem-se secoes com mesma area.

(b) Supondo conhecidas as expressoes do volume do cone e do cilindro, prove que o volume de uma esfera de raio

R e dado por4

3πR3.

Solucao

(a) Vamos denotar por x a distancia do plano ao vertice V dos cones e ao centro O da esfera. Suponhamos inicialmente o

plano abaixo de V e O.

Como na figura, os triangulos V C1P1 e V C2P2 sao semelhantes, logo

C1P1

C2P2

=V C1

V C2

.

Como V C1 = x, C2P2 = R e V C2 = R, temos

C1P1

R=x

R∴ C1P1 = x.

Page 69: MA13 { Geometria { AV1 { 2014 - professor.ufabc.edu.brprofessor.ufabc.edu.br/~jair.donadelli/MA13/MA13.pdf · determine a soma dos quadrados das medidas das diagonais AC e BD em fun˘c~ao

Com isso, a secao do solido S e a coroa circular entre os cırculos de raios x e R, logo, sua area e dada por

SessaoS = πR2 − πx2 = π(R2 − x2).

A secao da esfera pelo plano sera o cırculo de raio CP3 da figura. Como o triangulo OC3P3 e reto em C3, temos

OP32

= CP32

+OC32,

e, como OC3 = x e OP3 = R,

R2 = CP32

+ x2,

logo

CP32

= R2 − x2.

Com isso, a area da secao da esfera pelo plano sera dada por

Sessaoesfera = π(R2 − x2) = SessaoS ,

como querıamos provar.

O raciocınio e identico para planos acima do vertice e do centro da esfera, bastando alterar a figura.

Para o plano que passa pelo vertice dos cones e pelo centro da esfera, as secoes em ambos os solidos serao cırculos de

raio R e, assim, terao mesma area.

(b) Pelo Princıpio de Cavalieri, a esfera tera o mesmo volume do solido S. Mas este solido tem volume dado pelo volume

do cilindro de base de raio R e altura 2R, subtraıdo de dois cones solidos de base de raio R e altura R. Assim,

Volesfera = π ·R2 · 2R− 2

(π ·R2 ·R

3

)= 2πR3 − 2πR3

3=

6πR3 − 2πR3

3=

4

3πR3.

Questao 02 [ 2,00 ]

Em um tetraedro ABCD, AB = x, CD = y e as demais arestas medem z. Determine a distancia entre as arestas

AB e CD em funcao de x, y e z.

Solucao

Sejam M e N pontos medios de AB e CD, respectivamente.

Os triangulos ACB e ADB sao congruentes (LLL) e isosceles, logo, as alturas CM e DM sao congruentes. Assim, o triangulo

DMC e isosceles, e, como N e ponto medio de CD, MN sera altura de DMC. Da mesma forma, MN e altura do triangulo

isosceles ABN . Com isso, MN e o segmento da perpendicular comum as arestas AB e CD, logo a distancia entre estes

segmentos e MN . Vamos entao calcular este comprimento.

O triangulo AMD e retangulo em M , logo, denotando h = DM ,

z2 =(x

2

)2+ h2,

Page 70: MA13 { Geometria { AV1 { 2014 - professor.ufabc.edu.brprofessor.ufabc.edu.br/~jair.donadelli/MA13/MA13.pdf · determine a soma dos quadrados das medidas das diagonais AC e BD em fun˘c~ao

portanto

h2 = z2 − x2

4.

Denotando d = MN , como DNM e um triangulo retangulo em N , temos

h2 = d2 +(y

2

)2,

logo

d2 = h2 −(y

2

)2= h2 − y2

4,

e entao

d2 = z2 − x2

4− y2

4.

Com isso,

d =

√z2 − x2

4− y2

4.

Questao 03 [ 2,00 ]

Sobre o lado BC de um quadrado ABCD marcam-se os pontos E e F tais queBE

BC=

1

3eCF

BC=

1

4. Os segmentos

AF e ED intersectam-se em P . Determine a que fracao da area do quadrado ABCD corresponde a area do triangulo

BPE.

Solucao

Seja a a medida da aresta do quadrado. Assim,

BE =1

3BC =

a

3

e

CF =1

4BC =

a

4.

Como EFP = DAP , FEP = ADP (alternos internos nos dois casos) e EPF = DPA (opostos pelo vertice), os triangulos

APD e FPE sao semelhantes. Denotando por h e H as alturas desses triangulos, relativas as lados EF e DA, respectivamente,

temosh

H=EF

DA.

Como DA = a e

EF = BC −BE − CF = a− a

3− a

4=

5a

12,

temos entaoh

H=

5a12

a=

5

12,

e, portanto,

h =5

12H.

Page 71: MA13 { Geometria { AV1 { 2014 - professor.ufabc.edu.brprofessor.ufabc.edu.br/~jair.donadelli/MA13/MA13.pdf · determine a soma dos quadrados das medidas das diagonais AC e BD em fun˘c~ao

Mas h+H = a, logo H = a− h e entao

h =5

12(a− h),

logo

h+5

12h = a,

que implica17

12h =

5

12a

ou ainda

h =5

17a.

Assim, a area de BPE e

S =1

2·BE · h =

1

2· a

3· 5

17a =

5

102a2.

Como a area de ABCD e a2, a area do triangulo BPE e 5102

da area de ABCD.

Questao 04 [ 2,00 ]

Um poliedro convexo com 32 vertices possui apenas faces triangulares. Determine o numero de arestas e faces deste

poliedro.

Solucao

Vamos denotar por F o numero de faces e A o numero de arestas. Como este poliedro tem apenas faces triangulares, temos

2A = 3F

(isto e, cada face contabiliza 3 arestas, sendo que, cada aresta e contada duas vezes, uma para cada face em que esta contida).

Com isso,

A =3

2F.

Pelo Teorema de Euler, temos

32−A+ F = 2,

logo

32− 3

2F + F = 2

e, entao,

−F2

= −30,

implicando F = 60.

Com isso, temos

A =3

2· 60 = 90.

Questao 05 [ 2,00 ]

(a) Usando apenas a identidade fundamental da trigonometria e as formulas de arcos duplos prove que: cos(2x) =

2 cos2(x)− 1, para todo x real.

(b) Sabendo que cos(x2

)= 1

3 , calcule cos(x).

Page 72: MA13 { Geometria { AV1 { 2014 - professor.ufabc.edu.brprofessor.ufabc.edu.br/~jair.donadelli/MA13/MA13.pdf · determine a soma dos quadrados das medidas das diagonais AC e BD em fun˘c~ao

Solucao

(a) Usando a relacao fundamental da trigonometria e as formulas de arcos duplos temos que:

cos2(x) + sen2(x) = 1

e

cos(2x) = cos2(x)− sen2(x).

Somando as equacoes anteriores segue que:

cos(2x) + 1 = 2 cos2(x).

Logo, cos(2x) = 2 cos2(x)− 1.

(b) Usando o item (a) temos que:

cos(x) = cos

(2x

2

)= 2 cos2

(2x

2

)− 1 = 2

(1

3

)2

= −7

9.

Page 73: MA13 { Geometria { AV1 { 2014 - professor.ufabc.edu.brprofessor.ufabc.edu.br/~jair.donadelli/MA13/MA13.pdf · determine a soma dos quadrados das medidas das diagonais AC e BD em fun˘c~ao

MESTRADO PROFISSIONAL EM MATEMATICA EM REDE NACIONAL

AVF - MA13 - 2015.1 - Gabarito

Questao 01 [ 2,00 pts ]

Mostre que se uma reta e tangente a um cırculo entao ela e perpendicular ao raio que liga o centro do cırculo ao

ponto de tangencia.

Solucao

Considere o cırculo de centro O e seja T o ponto de tangencia da reta r ao cırculo. Designemos por P o pe da perpendicular

a reta r baixada por O. Observe que basta mostrar que P = T .

Suponha, por contradicao, que P e T sao distintos, assim o triangulo OPT e retangulo. Logo OT > OP , pois o maior lado

se opoe ao maior angulo. Como OT e o raio do cırculo, P e um ponto interior do cırculo.

Seja T′∈ r tal que PT = PT ′ com T

′= T . Daı, os triangulos OPT e OPT

′sao congruentes por LAL. Logo OT = OT ′ e

entao T′pertence ao cırculo. Contradicao, pois a reta r e tangente ao cırculo em T .

Portanto P = T .

Questao 02 [ 2,00 pts ]

Denotando por l1 e l2 as medidas dos lados nao paralelos de um trapezio, e por d1 e d2 as distancias do ponto de

intersecao das diagonais a estes lados, respectivamente, mostre que

d1d2

=l2l1.

Solucao

Sejam ABCD o trapezio, com lados nao paralelos AD e BC medindo l1 e l2 respectivamente, e P o ponto de intersecao das

diagonais.

Page 74: MA13 { Geometria { AV1 { 2014 - professor.ufabc.edu.brprofessor.ufabc.edu.br/~jair.donadelli/MA13/MA13.pdf · determine a soma dos quadrados das medidas das diagonais AC e BD em fun˘c~ao

Os triangulos ACD e BCD possuem a mesma base CD e alturas de mesma medida (pois AB e CD sao paralelos), logo

possuem a mesma area. Com isso,

Area(APD) = Area(ACD)− Area(PCD)

= Area(BCD)− Area(PCD) = Area(BPC).

Por outro lado,

Area(APD) =l1 · d12

e

Area(BPC) =l2 · d22

,

portantol1 · d12

=l2 · d22

.

Mas isto implica que

l1 · d1 = l2 · d2

e, consequentemente,d1d2

=l2l1.

Questao 03 [ 2,00 pts ]

Em uma folha de papel retangular, de medidas 10cm por 6cm, sao feitas as dobras paralelas AB e CD, como na

figura abaixo. Se DE e BF medem ambos 1 cm, determine a medida de AE.

Solucao

Seja G tal que EG e o lado da folha retangular que contem o ponto A e denote por G′ o ponto correspondente a G apos a

dobra, como na figura abaixo. Seja ainda P a projecao ortogonal de D sobre FG.

Page 75: MA13 { Geometria { AV1 { 2014 - professor.ufabc.edu.brprofessor.ufabc.edu.br/~jair.donadelli/MA13/MA13.pdf · determine a soma dos quadrados das medidas das diagonais AC e BD em fun˘c~ao

Denotando AE = x, teremos entao AG = AG′ = 6− x.

Como PG = DE = BF = 1cm, e como FG = 10cm temos PB = 8cm. E, como DP = EG = 6cm, aplicando o Teorema de

Pitagoras ao triangulo DPB, temos BD = 10cm. Alem disso, como BG′ = BG = 9cm, temos entao DG′ = 1cm.

Os triangulos DEA e DG′A serao ambos retangulos, com mesma hipotenusa DA e tais que DE = 1 = DG′. Estes triangulos

serao, portanto, congruentes. Com isso, x = AE = AG′ = 6− x, o que implica 2x = 6, logo AE = x = 3cm.

Questao 04 [ 2,00 pts ]

Considere um tetraedro ABCD e sejam M , N , P , Q, R e S os pontos medios das arestas AB, AC, AD, BC, CD e

BD, respectivamente. Prove que o volume do octaedro MNPQRS e metade do volume do tetraedro ABCD.

Solucao

O volume do octaedro MNPQRS e obtido retirando-se, do volume do tetraedro ABCD, os volumes dos tetraedros APMN ,

BMQS, CNRQ e DPRS.

Considere o tetraedro APMN . ComoM , N e P sao pontos medios de AB, AC e AD, respectivamente, APMN e semelhante

ao tetraedro ABCD, com razao1

2. Com isso,

V(APMN)

V(ABCD)=

(1

2

)3

=1

8.

Pelo mesmo raciocınio, os tetraedros BMQS, CNRQ e DPRS serao tambem semelhantes a ABCD com razao1

2, logo

V(BMQS)

V(ABCD)=

V(CNRQ)

V(ABCD)=

V(DPRS)

V(ABCD)=

1

8.

Com isso,

V(APMN) = V(BMQS) = V(CNRQ) = V(DPRS) =1

8V(ABCD),

e entao

V(MNPQRS) = V(ABCD)− V(APMN)− V(BMQS)− V(CNRQ)− V(DPRS)

= V(ABCD)− 4 · 18V(ABCD)

=1

2· V(ABCD).

Page 76: MA13 { Geometria { AV1 { 2014 - professor.ufabc.edu.brprofessor.ufabc.edu.br/~jair.donadelli/MA13/MA13.pdf · determine a soma dos quadrados das medidas das diagonais AC e BD em fun˘c~ao

Questao 05 [ 2,00 pts ]

Determine o cosseno do angulo entre duas faces de um octaedro regular que possuam uma aresta em comum.

Solucao

Sejam A e B dois vertices opostos do octaedro, e M o ponto medio de uma das arestas do octaedro que nao tenha A ou B

como extremo, como na figura abaixo.

A medida do angulo entre as faces sera dado por AMB.

Denote por a a medida da aresta do octaedro. Sendo O o centro do octaedro, O sera o centro da secao o octaedro destacada

na figura, que e um quadrado. Assim, MO = a2.

O segmento MA sera a altura da face que o contem, ou seja, a altura de um triangulo equilatero de lado a. Assim,

MA = a√

32

. Da mesma forma, MB = a√3

2.

O triangulo AOM e retangulo em O. Por Pitagoras, temos entao

OA2+OM

2= MA

2,

que implica

OA2+

(a2

)2

=

(a√3

2

)2

,

e entao

OA2=

3a2

4− a2

4.

Com isso,

OA =a√2

2.

Como OB ≡ OA, temos

AB = OA+OB = 2OA = a√2.

O triangulo AMB tera entao lados de medidas AB = a√2 e AM = BM = a

√3

2. Pela Lei dos Cossenos, temos entao

AB2= AM

2+BM

2 − 2AM BM cos(AMB

),

logo (a√2)2

=

(a√3

2

)2

+

(a√3

2

)2

− 2

(a√3

2

)2

cos(AMB

),

e entao

2a2 =3a2

2− 3a2

2cos

(AMB

).

Cancelando o a2 e simplificando a igualdade acima, temos

cos(AMB

)= −1

3.

Page 77: MA13 { Geometria { AV1 { 2014 - professor.ufabc.edu.brprofessor.ufabc.edu.br/~jair.donadelli/MA13/MA13.pdf · determine a soma dos quadrados das medidas das diagonais AC e BD em fun˘c~ao

MESTRADO PROFISSIONAL EM MATEMATICA EM REDE NACIONAL

AVF - MA13 - 2016.1 - Gabarito

Questao 01 [ 2,00 pts ]

Em um triangulo ABC de perımetro 9, o lado BC mede 3 e a distancia entre os pes das bissetrizes interna e externa

partindo de A e igual a 4. Calcule as medidas dos lados AB e AC do triangulo.

Solucao

Sejam P e Q os pes das bissetrizes interna e externa, respectivamente, partindo de A. Denotando x = AB, temos AC =

2P −AB −BC = 9− x− 3 = 6− x. Pelo Teorema das Bissetrizes Internas,

BP

AB=

PC

AC=

BP + PC

AB + AC,

logoBP

x=

PC

6− x=

3

6.

Com isso,

BP =x

2e PC =

6− x

2.

Como QP = 4, temos

QB = QP −BP = 4− x

2

e

QC = QP + PC = 4 +6− x

2.

Pelo Teorema das Bissetrizes Externas,QB

AB=

QC

AC,

logo

4− x

2x

=4 +

6− x

26− x

.

Simplificando,4

x− 1

2=

4

6− x+

1

2,

e entao6− 2x

x(6− x)=

1

4.

Desta forma, temos x2 − 14x + 24 = 0, implicando x = 2 ou x = 12.

Repare, porem, que AB = x = 12 nao e possıvel, pois este valor e maior que o perımetro do triangulo. Assim, AB = 2, o

que implica AC = 6−AB = 4.

Esta solucao considerou o vertice B entre os pes das bissetrizes relativas a A. Se C estivesse entre os pes das bissetrizes,

terıamos AB = 4 e AC = 2.

Page 78: MA13 { Geometria { AV1 { 2014 - professor.ufabc.edu.brprofessor.ufabc.edu.br/~jair.donadelli/MA13/MA13.pdf · determine a soma dos quadrados das medidas das diagonais AC e BD em fun˘c~ao

Questao 02 [ 2,00 pts ]

Considere um triangulo ABC retangulo em A. Por A, trace uma reta r perpendicular a BC, que encontra BC em

H. Em r, marque o ponto P , tal que AP = BC, estando A entre P e H. Trace os segmentos PB e PC.

(a) Prove que a area do quadrilatero PBAC eBC

2

2.

(b) Considere o quadrado ABDE tal que A esta entre E e C. Prove que o triangulo BCD e congruente ao triangulo

APB.

(c) Prove que area do triangulo BCD eAB

2

2.

Solucao

(a) Observe que Area(PBAC) = Area(APB) + Area(APC).

Como BH e a altura do triangulo APB relativa ao lado AP , temos

Area(APB) =BH ·AP

2=

BH ·BC

2

e, como CH e a altura do triangulo APC relativa ao lado AP , temos

Area(APC) =CH ·AP

2=

CH ·BC

2.

Assim,

Area(PBAC) = Area(APB) + Area(APC) =BH ·BC

2+

CH ·BC

2=

BC(BH + CH

)2

=BC

2

2.

(b) Temos CAH = ABC, pois ambos sao complementares ao angulo ACB, e PAE = CAH, pois os angulos sao opostos

pelo vertice. Com isso, PAE = ABC. Assim, PAB = CBD, pois ambos os angulos sao a soma de ABC com um angulo

reto.

Page 79: MA13 { Geometria { AV1 { 2014 - professor.ufabc.edu.brprofessor.ufabc.edu.br/~jair.donadelli/MA13/MA13.pdf · determine a soma dos quadrados das medidas das diagonais AC e BD em fun˘c~ao

Sabemos tambem que BC = AP (dado do problema) e BD = AB (pois ABDE e um quadrado), logo, pelo caso LAL,

os triangulos BCD e APB sao congruentes.

(c) Como, pelo item (b), BCD e APB sao congruentes, temos

Area(BCD) = Area(APB) =BH ·BC

2,

como calculado no item (a). Por outro lado, uma das relacoes metricas do triangulo retangulo nos diz que BH · BC =

AB2, logo

Area(BCD) =AB

2

2.

Questao 03 [ 2,00 pts ]

OABC e um tetraedro tal que AOB = AOC = BOC = 90◦, onde OA = x, OB = y e OC = z. Sabe-se que

AB =√

41, AC = 2√

13 e BC =√

61.

(a) Determine x, y e z.

(b) Determine o volume do tetraedro ABCO.

Solucao

(a) Aplicando o Teorema de Pitagoras nos triangulos ABO, ACO e BCO, todos eles retangulos em O, temos:

x2 + y2 = AB2

= 41. (I)

x2 + z2 = AC2

= 52. (II)

y2 + z2 = BC2

= 41. (III)

Somando membro a membro as tres equacoes anteriores, temos:

2x2 + 2y2 + 2z2 = 154

e entao

x2 + y2 + z2 = 77 (IV)

Das subtracoes abaixo, obtemos os valores desejados:

(IV)− (III)⇒ x2 = 16 ∴ x = 4cm.

(IV)− (II)⇒ y2 = 25 ∴ y = 5cm.

(IV)− (I)⇒ z2 = 36 ∴ z = 6cm.

(b) Tomando o triangulo ABO como base do tetraedro, OC sera sua altura. Daı, seu volume e:

V =1

3· x · y

2· z =

1

3· 4 · 5

2· 6 ∴ V = 20cm3.

Questao 04 [ 2,00 pts ]

Sejam L um cırculo de raio 1 de centro O e r uma reta (no mesmo plano que L) exterior a L. Diga como construir,

com regua e compasso, uma reta s tangente ao cırculo e que determina, com r, um angulo de 45◦.

Solucao

A solucao abaixo nao depende das medidas dadas.

Page 80: MA13 { Geometria { AV1 { 2014 - professor.ufabc.edu.brprofessor.ufabc.edu.br/~jair.donadelli/MA13/MA13.pdf · determine a soma dos quadrados das medidas das diagonais AC e BD em fun˘c~ao

Uma possıvel construcao esta representada acima e descrita abaixo:

1. Trace a reta perpendicular a r passando por O e seja P sua intersecao com r.

2. Construa o cırculo de centro P e raio PO e seja A uma das intersecoes deste cırculo com r. Repare que, por construcao,

o triangulo APO e retangulo em P e isosceles, com OP ≡ AP . Com isso, PAO = 45◦.

3. Trace a reta t determinada por A e O e seja B a intersecao de t com o cırculo dado, de forma que O esteja entre A e B.

4. Construa o cırculo de centro B e raio BA, e seja C seu ponto intersecao com r diferente de A. Note que CB ≡ AB,

logo ABC e isosceles, com ACB = CAB = 45◦. Teremos ainda que ABC = 90◦.

5. A reta s procurada e a determinada por B e C.

Questao 05 [ 2,00 pts ]

Considere tres retas r, s e t do espaco tais que qualquer plano seja concorrente a pelo menos uma destas retas.

Considere ainda um poliedro tal que

• todas as suas faces sao quadrilateros;

• cada uma de suas arestas e paralela a alguma das retas r, s ou t; e

• se um plano contem uma das faces, nenhum vertice do poliedro pode estar neste plano, alem dos vertices da

propria face.

Prove que

(a) todas as faces deste poliedro sao paralelogramos, e

(b) este poliedro e um prisma.

Solucao

Page 81: MA13 { Geometria { AV1 { 2014 - professor.ufabc.edu.brprofessor.ufabc.edu.br/~jair.donadelli/MA13/MA13.pdf · determine a soma dos quadrados das medidas das diagonais AC e BD em fun˘c~ao

(a) Considere uma das faces do poliedro, a qual chamaremos de ABCD. Sem perda de generalidade, digamos que AB ‖ r.

Nao poderemos ter BC ‖ r, pois, neste caso, teremos AB e BC consecutivos e colineares. Digamos entao, sem perda de

generalidade, que BC ‖ s.

Observe que ja concluımos que cada uma das retas r e s e paralela ou esta contida no plano da face ABCD. Assim,

pela informacao dada sobre as retas, t deve ser concorrente a tal plano, logo nao pode ser paralela a este plano nem

pode estar contida nele. Portanto, nenhuma das arestas CD e AD desta face pode ser paralela a reta t.

Nao poderemos ter CD ‖ s (senao BC e CD seriam colineares) nem, como ja vimos, CD ‖ t. Portanto, CD ‖ r. Da

mesma forma, AD nao pode ser paralela a r, logo AD ‖ s.

Ate aqui, ja vimos que ABCD e um paralelogramo. Mas a mesma argumentacao vale para qualquer uma das faces do

poliedro, portanto todas as suas faces sao paralelogramos.

(b) Seja ABCD uma das faces. Pelo item (a), podemos supor, sem perda de generalidade, AB e CD paralelas a r e BC e

AD paralelas a s.

Considere uma aresta que tenha A, B, C ou D como vertice, e que nao esteja na face ABCD. Chamemos esta aresta

de EA. Como E nao pertence ao plano de ABCD, EA nao e paralela a r ou s, logo EA ‖ t.

Mas isto vale para todas as arestas que tem um vertice na face ABCD, portanto todas estas arestas sao paralelas a t, e

de cada vertice A, B, C e D sai apenas uma aresta que nao esta na face ABCD.

A face que tem A, B e E como tres de seus vertices devera ser um paralelogramo (item (a)), portando, seu quarto

vertice F e tal que EF ‖ AB e BF ‖ AE.

A face adjacente a ABFE, compartilhando a aresta BF , devera ser entao um paralelogramo BCGF , com FG ‖ BC. A

face adjacente a BCGF , compartilhando a aresta CF , devera ser entao um paralelogramo CDHG, com GH ‖ CD ‖ AB.

Por fim, teremos a face ADHE, com HE ‖ AD ‖ BC.

Repare que EFGH e, entao, um paralelogramo, pois EF e GH sao paralelos a AB, e FG e EH sao paralelos a BC.

Temos ainda que EFGH devera ser uma face. De fato, se nao fosse assim, haveria uma aresta EI, FI, GI ou HI que

nao estaria no plano de EFGH, portanto nao seria paralela a r e s, e nem poderia ser paralela a t, pois, neste caso,

terıamos duas arestas colineares.

Assim, temos o prisma ABCDEFGH, tal que ABCD e EFGH sao bases paralelas, com AE ‖ BF ‖ CG ‖ DH.

Page 82: MA13 { Geometria { AV1 { 2014 - professor.ufabc.edu.brprofessor.ufabc.edu.br/~jair.donadelli/MA13/MA13.pdf · determine a soma dos quadrados das medidas das diagonais AC e BD em fun˘c~ao

MA13 Geometria I - GABARITO DA AVALIACAO 2 - 2012/2

Questao 1. (pontuacao: 2)

No setor AOB de centro O, raio OA = 3 e angulo AOB = 60o esta inscrita uma circunferencia como mostra a figura.

a) Calcule o raio dessa circunferencia.

b) Calcule a area da regiao sombreada.

Uma solucao

A O

B

C M

D

r

r

a) Seja M o ponto medio do arco AB. O raio OM passa pelo centro C da circunferencia inscrita no setor. Seja CD

perpendicular a OA como mostra a figura acima e seja r = CD = DM o raio da circunferencia. Como AOM = 30o

entao, no triangulo ODC tem-se OC = 2r e, portanto, OM = 3r = 3, ou seja, r = 1 .

b) A area sombreada (S) e igual a area do setor AOM subtraıda da area do triangulo ODC e da area do setor

DCM do cırculo de centro C.

A area do setor AOM e π32

12 = 3π4 .

Como CD = 1 e OC = 2, entao OD =√

3 e a area do triangulo ODC e OD.OC2 =

√32 . Por outro lado, o angulo

DCM mede 120o e, portanto, o setor DCM do cırculo de centro C tem area igual a terca parte da area do cırculo

de centro C, ou seja, π3 . Assim, a area sombreada e S = 3π4 −

√32 −

π3 , ou seja, S = 5π−6

√3

12 .

Questao 2. (pontuacao: 2)

O Teorema das Tres Perpendiculares tem o seguinte enunciado:

“A reta r e perpendicular ao plano α no ponto A. A reta s esta contida em α e nao passa por A. O ponto B da

reta s e tal que AB e perpendicular a s. Entao, se P e qualquer ponto de r, PB e perpendicular a s.”

a) Faca uma figura que descreva o enunciado do Teorema.

b) Demonstre o Teorema.

Page 83: MA13 { Geometria { AV1 { 2014 - professor.ufabc.edu.brprofessor.ufabc.edu.br/~jair.donadelli/MA13/MA13.pdf · determine a soma dos quadrados das medidas das diagonais AC e BD em fun˘c~ao

Uma solucao

a)

A B

P

r

s

b) Como r e perpendicular a α entao r e ortogonal a qualquer reta de α , portanto r e ortogonal a s. Porem,

AB e perpendicular a s. Assim, s e ortogonal a duas retas concorrentes: AB e r. Logo s e perpendicular ao plano

determinado por AB e r, que chamaremos de plano β. Como P e B sao pontos de β entao s e perpendicular a PB,

como querıamos demonstrar.

Questao 3. (pontuacao: 2)

Em um cubo, ABCD e EFGH sao faces opostas e AE, BF , CG e DH sao arestas paralelas. Sejam M e N os

pontos medios das arestas BC e DH, respectivamente.

a) Se a aresta do cubo mede 2, calcule a distancia entre os pontos M e N .

b) Calcule o cosseno do angulo entre as retas AB e NM .

Uma solucao

A B

C D

E F

G H

M

N

O

a) Observe a figura acima. No triangulo CDN , retangulo em D, CD = 2 e DN = 1. Consequentemente, NC =√

5.

Como a aresta BC e perpendicular a face DCGH, o triangulo MCN e retangulo em C. Daı, MN2 = NC2 +MC2 =

5 + 1 = 6, ou seja MN =√

6.

b) Facamos uma translacao do segmento MN de forma que o ponto M concida com o ponto B. Nessa translacao,

o ponto N coincidira com o ponto O, centro da face ADHE. O angulo entre as retas reversas AB e NM e o angulo

entre as concorrentes AB e OB, ou seja, o angulo ABO = θ.

Page 84: MA13 { Geometria { AV1 { 2014 - professor.ufabc.edu.brprofessor.ufabc.edu.br/~jair.donadelli/MA13/MA13.pdf · determine a soma dos quadrados das medidas das diagonais AC e BD em fun˘c~ao

No triangulo ABO, temos AB = 2, OB = NM =√

6 e AO e a metade da diagonal do quadrado ADHE, ou seja,

AO =√

2.

A lei dos cossenos no triangulo ABO fornece:

(√

2)2 = 22 + (√

6)2 − 2.2.√

6.cos θ

Daı, encontramos cos θ =√63 . Isto tambem pode ser obtido notando-se que o triangulo ABO e retangulo em A e

usando-se diretamente a definicao de cosseno.

Questao 4. (pontuacao: 2)

O trapezio ABCD tem bases AB e CD. A altura do trapezio mede 8. As bases medem AB = 10 e CD = 6. As

diagonais AC e BD do trapezio dividiram o trapezio em quatro triangulos. Calcule as areas dos quatro triangulos

em que o trapezio ficou dividido.

Uma solucao

A B

C D

x

8 – x

P

10

6

8

Notacao: (XY Z) representa a area do triangulo de vertices X, Y e Z.

Seja P o ponto de intersecao das diagonais e seja x a distancia de P a base menor do trapezio. Como os triangulos

PAB e PCD sao semelhantes temos 610 = x

8−x , o que da x = 3 . Assim, (PCD) = 6.32 = 9 e (ABC) = 10.5

2 = 25.

(DAP ) = (DAB)− (PAB) =10.8

2− 25 = 40− 25 = 15

(CPB) = (CAB)− (PAB) = 40− 25 = 15

As areas dos quatro triangulos estao na figura abaixo:

15 15

25

9

Page 85: MA13 { Geometria { AV1 { 2014 - professor.ufabc.edu.brprofessor.ufabc.edu.br/~jair.donadelli/MA13/MA13.pdf · determine a soma dos quadrados das medidas das diagonais AC e BD em fun˘c~ao

Questao 5. (pontuacao: 2)

No cubo ABCDA′B′C ′D′ de aresta a, os pontos M , N , P e Q sao medios das arestas A′B′, B′C ′, C ′D′ e A′D′,

respectivamente. Foram feitas as secoes pelos planos AMQ, BNM , CPN e DPQ. Retirando-se os quatro tetraedros

formados, resultou o poliedro P ilustrado na Figura 1. O poliedro P possui duas bases paralelas e faces laterais

triangulares. Ele e um prismatoide.

a) Calcule o volume do poliedro P .

A B

C D

A' B'

C' D'

M N

P

Q

Figura 1

Observe agora a Figura 2; pelo ponto medio X da aresta AA′ foi tracado um plano paralelo a face ABCD que

determinou em P uma secao octogonal. A forma dessa secao equidistante das bases do poliedro P , que e chamada

se secao media, esta ilustrada na Figura 3.

No poliedro P , representaremos a area da base ABCD por S, a area da base MNPQ por s, a area da secao media

por Sm e a distancia entre as bases por h.

b) Calcule a area da secao media e calcule o volume de P usando a formula do volume dos prismatoides: V =h6 (S + s+ 4Sm) .

Figura 2 Figura 3 A B

C D

A' B'

C' D'

M N

P Q

X

X

Page 86: MA13 { Geometria { AV1 { 2014 - professor.ufabc.edu.brprofessor.ufabc.edu.br/~jair.donadelli/MA13/MA13.pdf · determine a soma dos quadrados das medidas das diagonais AC e BD em fun˘c~ao

Uma solucao

a) Um dos tetraedros retirados e AAMQ. Sua base e o triangulo retangulo AMQ de catetos AM = AQ = a2 e

altura AA = a . O volume desse tetraedro e 13 .

(a/2)(a/2)2 .a = a3

24 . Como quatro desses tetraedros foram retirados, o

volume do poliedro P e V = a3 − 4.a3

24 = a3 − a3

6 = 5a3

6 .

b) A secao media e obtida de um quadrado XY ZW , congruente com ABCD retirando-se quatro triangulos

retangulos isosceles congruentes. O plano da secao media corta a aresta BB do cubo em Y e corta as arestas MA e

MB do poliedro P em E e F , respectivamente.

X Y E F a/2

Temos EF = AB2 = a

2 (ja que a reta XY une pontos medios de lados do triangulo AMB) e, consequentemente,

XE = FY = a4 . Assim, cada um dos pequenos triangulos retangulos tem area 1

2 .a4 .a4 = a2

32 e a area da secao media e

Sm = a2 − 4.a2

32 = 7a2

8 .

Aplicando a formula do volume do prismatoide temos:

V =a

6(a2 +

a2

2+ 4.

7a2

8) =

a

6(a2 +

a2

2+

7a2

2) =

a

6.10a2

2=

5a3

6

o que coincide com o resultado do item a).

Page 87: MA13 { Geometria { AV1 { 2014 - professor.ufabc.edu.brprofessor.ufabc.edu.br/~jair.donadelli/MA13/MA13.pdf · determine a soma dos quadrados das medidas das diagonais AC e BD em fun˘c~ao

GABARITO MA13 Geometria I - Avaliacao 3 - 2012/2

A area de um triangulo ABC sera denotada por (ABC).

Questao 1. (pontuacao: 2)

A figura abaixo mostra as semirretas perpendiculares r e s, tres circunferencias pequenas cada uma com raio igual

a 1 e uma circunferencia grande de centro O. Uma das circunferencias pequenas e tangente a r e a s, cada uma das

outras duas e tangente a ela e a uma das semirretas, e a circunferencia grande e tangente as semirretas e a duas das

circunferencias pequenas.

r

s

O

Calcule o raio da circunferencia grande.

Uma solucao:

Na figura a seguir, A e o centro da circunferencia pequena que tangencia r e a circunferencia grande, OT e

perpendicular a r e a reta BC passa por A e e paralela a r.

A B

O

C

r T

s R

Page 88: MA13 { Geometria { AV1 { 2014 - professor.ufabc.edu.brprofessor.ufabc.edu.br/~jair.donadelli/MA13/MA13.pdf · determine a soma dos quadrados das medidas das diagonais AC e BD em fun˘c~ao

Seja R o raio da circunferencia grande. No triangulo retangulo ABO temos OA = R+1, OB = R−1 e AB = R−3.

O teorema de Pitagoras conduz a equacao R2 − 10R+ 9 = 0 cujas raızes sao 1 e 9.

Devido as caracterısticas do problema, a menor raiz e o raio da circunferencia pequena tangente as duas semirretas

e a maior raiz e o raio da circunferencia grande.

O raio da circunferencia grande e igual a 9.

Questao 2. (pontuacao: 2)

No triangulo ABC a bissetriz do angulo BAC encontra o lado BC em D.

a) Prove que DBDC = AB

AC (teorema da bissetriz interna).

b) Use o teorema acima e a figura abaixo para calcular a tangente de 15o.

15o 15o

2

x

Uma solucao:

a) Se dois triangulos tem mesma altura, entao a razao entre suas areas e igual a razao entre suas bases. Assim,

(ADB)

(ADC)=DB

DC

A

B C D

E

F

d d

Page 89: MA13 { Geometria { AV1 { 2014 - professor.ufabc.edu.brprofessor.ufabc.edu.br/~jair.donadelli/MA13/MA13.pdf · determine a soma dos quadrados das medidas das diagonais AC e BD em fun˘c~ao

Sejam DE e DF perpendiculares a AB e AC como na figura anterior. Como todo ponto da bissetriz de um angulo

equidista dos lados desse angulo, entao DE = DF = d. Assim,

(ADB)

(ADC)=

(1/2).AB.d

(1/2).AC.d=AB

AC, portanto

DB

DC=AB

AC, c.q.d.

b) Como o triangulo ABC da figura e retangulo em B e tem angulo BAC = 30o e hipotenusa AC = 2, entao

BC = 1 e AB =√

3.

15o 15o

2

x

A B

C

D

3

1 – x

O teorema da bissetriz interna aplicado a esse triangulo fornece: DBDC = AB

AC , ou seja, x1−x =

√32 .

Aplicando propriedades das proporcoes podemos escrever:

x√3

=1− x

2=x+ (1− x)√

3 + 2=

1

2 +√

3.

Porem, observando o triangulo ABD, vemos que x√3

e a tangente do angulo de 15o. Assim,

tan15o =1

2 +√

3= 2−

√3.

Questao 3. (pontuacao: 3)

O losango ABCD tem lado 3 e angulo A = 60o. Os pontos M , N , P e Q pertencem aos lados AB, BC, CD e DA,

respectivamente e sao tais que AM = BN = CP = DQ = 1 .

a) Justifique, de forma breve, porque o quadrilatero MNPQ e um paralelogramo.

b) Calcule a area do quadrilatero MNPQ.

c) Calcule a distancia entre os pontos M e P .

Page 90: MA13 { Geometria { AV1 { 2014 - professor.ufabc.edu.brprofessor.ufabc.edu.br/~jair.donadelli/MA13/MA13.pdf · determine a soma dos quadrados das medidas das diagonais AC e BD em fun˘c~ao

Uma solucao:

A

B

C

D

M N

P Q

a) Os triangulos AMQ e CPN sao congruentes (caso LAL). Daı, MQ = NP . Os triangulos BNM e DQP sao con-

gruentes (caso LAL); daı, MN = QP . Assim, o quadrilatero MNPQ possui dois pares de lados opostos congruentes.

Logo, e um paralelogramo.

b) (AMQ) = 12 .AM.AQ.sen 60o = 1

2 .1.2.√32 =

√32 .

Como os angulos de 60o e 120o possuem mesmo seno, concluımos que os triangulos AMQ, BNM , CPN e DQP

possuem todos a mesma area, igual a√32 .

A area do losango e igual a AB.AD.sen 60o = 3.3.√32 = 9

√3

2 .

A area do paralelogramo e

S =9√

3

2− 4

√3

2=

5√

3

2.

c) Seja R o ponto medio de PD. Como AM e paralelo a DR e ambos tem comprimento 1 entao AMRD e um

paralelogramo e MR = AD = 3. Alem disso, RP = 1 e MRP = 120o .

A

M

P R

D

B

C

No triangulo MRP a lei dos cossenos fornece:

MP 2 = MR2 +RP 2 − 2.MR.RP.cos 120o = 32 + 12 − 2.3.1.(−1

2) = 13 =⇒ MP =

√13.

Page 91: MA13 { Geometria { AV1 { 2014 - professor.ufabc.edu.brprofessor.ufabc.edu.br/~jair.donadelli/MA13/MA13.pdf · determine a soma dos quadrados das medidas das diagonais AC e BD em fun˘c~ao

Questao 4. (pontuacao: 1)

O icosaedro regular e o poliedro formado por 20 faces triangulares equilateras. Determine quantas diagonais do

icosaedro nao passam pelo seu centro.

Uma solucao:

O icosaedro possui 20 faces triangulares. Como cada aresta e lado de exatamente duas faces, o numero de arestas

do icosaedro e A = 20.32 = 30.

O numero de vertices pode ser calculado pela relacao de Euler V −A+ F = 2, de onde V = 12.

Cada segmento que une dois vertices do icosaedro ou e aresta ou e diagonal. Assim, denotando por C2V o numero

de escolhas de subconjuntos com dois elementos do conjunto de todos os vertices do icosaedro, o numero de diagonais

do icosaedro e

D = C2V −A = C2

12 −A =12!

10!2!− 30 = 66− 30 = 36

O icosaedro possui 6 pares de vertices diametralmente opostos e cada diagonal que une dois vertices diametralmente

opostos passa pelo centro do icosaedro. Essas sao as unicas diagonais que passam pelo centro. Entao, o numero de

diagonais que nao passam pelo centro e 36− 6 = 30.

Questao 5. (pontuacao: 2)

Considere o paralelepıpedo retangulo de bases ABCD e EFGH e com arestas laterais AE, BF , CG e DH. As

medidas sao AB = 6, AD = AE = 4 e M e o ponto medio da aresta EF . Sao feitas as secoes pelos planos MHA e

MBG. Retirando-se os tetraedros EMHA e FMBG resulta o poliedro P.

a) Faca um desenho do poliedro P e calcule seu volume.

b) Determine o cosseno do angulo entre as retas AH e MG.

Page 92: MA13 { Geometria { AV1 { 2014 - professor.ufabc.edu.brprofessor.ufabc.edu.br/~jair.donadelli/MA13/MA13.pdf · determine a soma dos quadrados das medidas das diagonais AC e BD em fun˘c~ao

Uma solucao:

A B

C D

E F

G H

M

O desenho de P esta acima.

Se pensarmos o tetraedro EMHA com base EMH e altura EA, podemos calcular seu volume do seguinte modo:

v =1

3.EM.EH

2.EA =

1

3.3.4

2.4 = 8

O tetraedro FMBG tem tambem volume v = 8 porque e congruente com EMHA.

O volume de P e o volume do paralelepıpedo subtraıdo dos volumes dos tetraedros, ou seja,

V = 6.4.4− 2.8 = 96− 16 = 80

b) O angulo entre as retas AH e MG e o angulo entre BG e MG, ou seja, o angulo θ = BGM . Como FM = 3 e

FB = FG = 4 temos, pelo teorema de Pitagoras, BG = 4√

2 e MB = MG = 5.

H

θ

A

C D

E F

G

M

N

5

5

B

2ට2

O triangulo MBG e isosceles. Entao, assinalando o ponto N medio do lado BG, temos que MN e perpendicular

a BG. Assim, no triangulo MNG,

cos θ =NG

MG=

2√

2

5.

Page 93: MA13 { Geometria { AV1 { 2014 - professor.ufabc.edu.brprofessor.ufabc.edu.br/~jair.donadelli/MA13/MA13.pdf · determine a soma dos quadrados das medidas das diagonais AC e BD em fun˘c~ao

GABARITO MA13 - Avaliacao Final 1o semestre - 2013

Questao 1. (pontuacao: 2)

No triangulo isosceles ABC tem-se AB = AC. Os pontos M , N e P dos lados AB, BC e CA sao tais que PM = PN .

Sendo PMA = α, NPC = β e MNB = θ mostre que

θ =α+ β

2

Uma solucao:

Sejam ABC = ACB = x e PMN = PNM = y.

O angulo AMN e externo do triangulo MBN . Logo, α+ y = x+ θ.

O angulo BNP e externo do triangulo PNC. Logo, β + x = y + θ.

Somando, temos α+ β = 2θ, ou seja, θ = α+β2 , cqd.

Questao 2. (pontuacao: 2)

Considere o triangulo ABC, retangulo em A, sendo BC = a e AC = b. Seja K1 a circunferencia de centro C que

passa por A. A circunferencia K2 tem centro P sobre o lado BC, e tangente externamente a K1 e e tangente ao lado

AB.

a) (1,0) Descreva como se pode determinar com regua e compasso o ponto P .

b) (1,0) Determine o raio da circunferencia K2 em funcao de a e b.

Obs.: os itens acima podem ser resolvidos de maneira independente.

Page 94: MA13 { Geometria { AV1 { 2014 - professor.ufabc.edu.brprofessor.ufabc.edu.br/~jair.donadelli/MA13/MA13.pdf · determine a soma dos quadrados das medidas das diagonais AC e BD em fun˘c~ao

Uma solucao:

a)

Seja D o ponto onde K1 corta BC. A perpendicular a BC por D e tangente a K1 e corta AB em E. A bissetriz

do angulo DEB corta BC em P . De fato, pela construcao acima, P e equidistante das retas ED e EB. Logo, a

circunferencia de centro P que passa por D e tangente a K1 e ao lado AB.

b)

Tracemos PG, perpendicular a AB e PF perpendicular a AC como na figura acima. Sejam PD = PG = x. Da

semelhanca dos triangulos CFP e CAB temos:

b− xb+ x

=b

a

Assim,

x =b(a− b)a+ b

.

Page 95: MA13 { Geometria { AV1 { 2014 - professor.ufabc.edu.brprofessor.ufabc.edu.br/~jair.donadelli/MA13/MA13.pdf · determine a soma dos quadrados das medidas das diagonais AC e BD em fun˘c~ao

Questao 3. (pontuacao: 2)

A figura a seguir mostra duas semicircunferencias com mesmo centro O e com raios OD = r e OA = 2r. Na semi-

circunferencia maior foi assinalado um ponto B e angulo AOB mede α radianos. O raio OB cortou a circunferencia

menor em C e a regiao R e a que esta sombreada (delimitada pelo arco AB, segmento BC, arco CD e segmento

DA) na figura.

a) (1,0) Calcule o perımetro de R em funcao de r e α.

b) (1,0) Calcule a area de R em funcao de r e α.

Uma solucao:

a) O comprimento de um arco de circunferencia e igual a medida do angulo central em radianos multiplicada pelo

raio. Assim o perımetro de R e:

P = 3r + α.2r + r + (π − α)r = (α+ π + 4)r

b) A area de um setor de angulo central α em radianos em um cırculo de raio r e αr2

2 . Assim a area da regiao R e:

A =α(2r)2

2+

(π − α)r2

2=

(3α+ π)r2

2

Questao 4. (pontuacao: 2)

A aresta da base de uma piramide reta de base quadrada mede 2 unidades e a esfera inscrita nessa piramide tem

raio r (0 < r < 1).

Page 96: MA13 { Geometria { AV1 { 2014 - professor.ufabc.edu.brprofessor.ufabc.edu.br/~jair.donadelli/MA13/MA13.pdf · determine a soma dos quadrados das medidas das diagonais AC e BD em fun˘c~ao

A B

C D

E

a) (1,0) Calcule o volume da piramide em funcao de r.

b) (1,0) Se, para cada valor de r (0 < r < 1 ), o volume da piramide e V (r), faca um esboco do grafico dessa

funcao.

Uma solucao:

a)

Seja O o centro da base ABCD da piramide de vertice E como mostra a figura ao lado. Seja M o ponto medio

da aresta BC. Seja K o ponto sobre a altura OE o centro da esfera inscrita na piramide. Assim, tracando KT

perpendicular a face EBC temos KO = KT = r. Seja h = OE a altura da piramide. Da semelhanca dos triangulos

ETK e EOM temos

KT

OM=KE

EM

ou seja,

r

1=

h− r√h2 + 1

Dessa relacao determinamos a altura da piramide h = 2r1−r2 .

Page 97: MA13 { Geometria { AV1 { 2014 - professor.ufabc.edu.brprofessor.ufabc.edu.br/~jair.donadelli/MA13/MA13.pdf · determine a soma dos quadrados das medidas das diagonais AC e BD em fun˘c~ao

O volume da piramide e

V =1

3.22.

2r

1− r2=

8r

3(1− r2)

.

b) A funcao que associa r e V e crescente. Quando r se aproxima de 0, temos que V se aproxima de 0. Porem,

quando r se aproxima 1 temos que V tende a +∞. Logo, o grafico de V (r) tem o seguinte aspecto:

1

Questao 5. (pontuacao: 2)

Um copo de plastico rıgido e espessura muito fina tem a forma de um tronco e cone com 8 cm de diametro na

boca, 6 cm de diametro no fundo e 12 cm de altura.

a) (1,0) Determine um valor aproximado para o volume do copo (ou seja, o numero inteiro de cm3 que melhor

aproxima o volume).

b) (1,0) Determine um valor aproximado para a area externa total do copo (ou seja, o numero inteiro de cm2 que

melhor aproxima a area externa).

Uma solucao:

a) Os raios das bases sao 4 cm, e 3 cm e a formula do volume do tronco de cone fornece o resultado:

V =πh

3.(R2 + r2 +Rr) =

π.12

3(16 + 9 + 12) ≈ 3, 14.4.37 ≈ 465 cm3

b) A geratriz do tronco de cone e igual a√

122 + 1 que e aproximadamente igual a 12. Nesse copo, a altura e quase

igual a geratriz.

A area do copo e a soma da area lateral com a area da base, isto e:

A = π(R+ r)g + πr2 ≈ 3, 14.[(4 + 3).12 + 32] ≈ 292 cm2